122
Internal Medicine- Cardiology A 62-year-old hypertensive smoker male presents to the hospital with 2-month history of retrosternal chest pain on minor exertion. On physical examination, his blood pressure was 130/90 mm Hg, other findings were also unremarkable. The ECG was showing ST- depression in the precordial leads consistent with angina. All of the following statements about this patient’s intracoronary adenosine levels and the effect of adenosine on the coronary are true except: 1) Adenosine represents the first line of defence in protecting the myocardium against decreased oxygen supply/demand ratio 2) alfa 1 -receptor stimulate adenylyl cyclase while alfa 2 -receptor inhibit this enzyme system 3) Adenosine is a potent coronary vasodilator 4) Degree of release of adenosine from myocardium is directly proportional to the degree of oxygen deprivation 5) The negative inotropic and chronotropic responses are mediated by alfa 2 -receptor The correct answer is choice E The earliest response to an inadequate blood supply to the myocardium is dilation of the coronary resistance vessels. Ischemia or hypoxia-induced vasodilation is mediated by the release of adenosine from the myocardial cells. Also, the degree of vasodilation and the release of adenosine from the heart are directly proportional to the degree of oxygen deprivation. Adenosine represents the first line of defense in protection of the myocardium against a decrease in the oxygen supply/demand ratio by reducing coronary resistance and maximizing coronary blood flow. Adenosine receptors are classified as A 1 , which inhibit adenylyl cyclase, and A 2 , that stimulate this enzyme system. In the heart, A 1 -receptors are found on cardiomyocytes and vascular smooth muscle, whereas A 2 -receptors are found on endothelium and vascular smooth muscle. Cardiac cells A 1 - receptors mediate the negative chronotropic, dromotropic and inotropic responses. A 1 -Receptors in the heart also appear to mediate the beneficial effects of adenosine in prolonging the time to ischemic contracture and preconditioning. Adenosine A 2 -receptors are apparently located on the coronary vessels, because intravascular A 2 -agonists are more potent coronary vasodilators than A 1 -agonists. 62-year old female patient with a history of prior MI is planned to undergo revascularization. Which of the following modalities can detect the infarct most accurately as well as determine the prognosis better than the standard left ventricular volume and ejection fraction? 1) Echocardiography 2) Coronary Angiography

Internal Medicine- Cardiology

Embed Size (px)

DESCRIPTION

Internal Medicine- Cardiology

Citation preview

Page 1: Internal Medicine- Cardiology

Internal Medicine- Cardiology

A 62-year-old hypertensive smoker male presents to the hospital with 2-month history of retrosternal chest pain on minor exertion. On physical

examination, his blood pressure was 130/90 mm Hg, other findings were also unremarkable. The ECG was showing ST- depression in the precordial

leads consistent with angina.

All of the following statements about this patient’s intracoronary adenosine levels and the effect of adenosine on the coronary are true except:

1) Adenosine represents the first line of defence in protecting the myocardium against decreased oxygen supply/demand ratio 2) alfa1-receptor stimulate adenylyl cyclase while alfa2-receptor inhibit this enzyme system 3) Adenosine is a potent coronary vasodilator 4) Degree of release of adenosine from myocardium is directly proportional to the degree of oxygen deprivation 5) The negative inotropic and chronotropic responses are mediated by alfa2-receptor

The correct answer is choice E

The earliest response to an inadequate blood supply to the myocardium is dilation of the coronary resistance vessels. Ischemia or hypoxia-induced vasodilation is mediated by the release of adenosine from the myocardial cells. Also, the degree of vasodilation and the release of adenosine from the heart are directly proportional to the degree of oxygen deprivation. Adenosine represents the first line of defense in protection of the myocardium against a decrease in the oxygen supply/demand ratio by reducing coronary resistance and maximizing coronary blood flow.

Adenosine receptors are classified as A1, which inhibit adenylyl cyclase, and A2, that stimulate this enzyme system. In the heart, A1-receptors are found on cardiomyocytes and vascular smooth muscle, whereas A2-receptors are found on endothelium and vascular smooth muscle. Cardiac cells A1-receptors mediate the negative chronotropic, dromotropic and inotropic responses. A1-Receptors in the heart also appear to mediate the beneficial effects of adenosine in prolonging the time to ischemic contracture and preconditioning. Adenosine A2-receptors are apparently located on the coronary vessels, because intravascular A2-agonists are more potent coronary vasodilators than A1-agonists.

62-year old female patient with a history of prior MI is planned to undergo revascularization.

Which of the following modalities can detect the infarct most accurately as well as determine the prognosis better than the standard left ventricular

volume and ejection fraction?

1) Echocardiography 2) Coronary Angiography

Page 2: Internal Medicine- Cardiology

3) Cardiac computed tomography 4) Contrast-enhanced cardiac MRI 5) Nuclear imaging

The correct answer is choice D Contrast-enhanced cardiac MRI (choice D) can detect myocardial infarction most accurately. The transmural extent of late gadolinium enhancement in regions of dysfunctional myocardium accurately predicts the chances of recovery after successful revascularization. In patients with a prior MI, estimation of the size of the periinfarct zone by CMR using the delayed enhancement technique provides incremental prognostic value beyond left ventricular volumes and ejection fraction. Contrast-enhanced cardiac MRI is also able to characterize the presence and size of microvascular obstruction from infarction, which indicates an adverse clinical outcome in post MI patients. Echocardiography (choice A) is useful in the diagnosis of myocardial ischemia. The ischemic myocardium appears as a region of disordered contraction. However, abnormal wall motion may not be noticed in small infarctions. It is also used for assessing the left ventricular function and ejection fraction which is useful in assessing the prognosis after MI. Echocardiography can also help evaluate patients with chest pain and a nondiagnostic ECG who are suspected of having an aortic dissection. The relative portability of echocardiographic equipment makes it ideal for use in hospitalized patients of MI or even in the emergency department. Areas of abnormal regional wall motion are observed almost universally in patients with MI. When the infarction is small, it cannot always be detected with echocardiography. Coronary angiography (choice B) is used for assessing the degree of occlusion or stenosis in the coronary arteries. It helps in deciding the treatment plan of the patient having coronary artery disease. Cardiac computed tomography (choice C) can provide useful cross-sectional information in patients with MI. In addition to the assessment of cavity dimensions and wall thickness, left ventricular aneurysms can be detected, and, of particular importance in patients with STEMI, intracardiac thrombi can be identified. Although cardiac computed tomography is a less convenient technique, it probably is more sensitive for thrombus detection than is echocardiography. Nuclear cardiac imaging (choice E) techniques can be useful for detecting MI; assessing infarct size, collateral flow, and jeopardized myocardium; determining the effects of the infarct on ventricular function; and establishing prognosis of patients with STEMI. However, application is limited due to necessity of moving a critically ill patient from the coronary care unit to the nuclear medicine department. Cardiac radionuclide imaging for the diagnosis

Page 3: Internal Medicine- Cardiology

of MI should be restricted to special situations when the triad of clinical history, ECG findings, and serum marker measurements is unavailable or unreliable.

A 55 year old woman presented with a 3 month history of an occasional jumping sensation in her chest. She had no prior history of cardiovascular

disease and did not take medication. Clinical examination of the cardiovascular system and a an Electrocardiogram (ECG) were normal. Subsequent 24

hour holter monitoring revealed 5 Ventricular premature beats. An exercise test and echocardiogram were normal.

Which of the following is the most appropriate management?

1) Reassurance 2) thallium exercise scanning 3) selective coronary arteriography 4) electrophysiologic testing 5) empiric drug therapy with procainamide

The correct answer is Choice A

Ventricular premature beats (ectopics) are common, particularly among older people and a large proportion of patients have no evidence of underlying heart disease. This arrhythmia may be exacerbated by physical or emotional stress, excess intake of caffeine, alcohol and smoking. Other causes include coronary artery disease, heart failure and heart valvular disorders. Frequent Ventricular premature beats in patients with a structural heart disorder may lead to more dangerous arrhythmias such as ventricular tachycardia or ventricular fibrillation, which can cause sudden death.

Choice A In an otherwise healthy person, no treatment is necessary other than reassurance, decreasing stress and avoiding caffeine, alcohol, and smoking.

Choice B Thallium exercise scanning assesses blood flow to the heart following injection of a radioactive tracer and gives similar information to an conventional exercise test. It can be performed in less mobile patients as the exercise part of the test can be performed pharmacologically. It is also significantly more expensive than an exercise test.

Choice C Coronary angiography is appropriate for patients with ventricular premature beats post myocardial infarction as subsequent angioplasty or

Page 4: Internal Medicine- Cardiology

coronary artery bypass surgery reduces the risk of fatal arrhythmias.

Choice D Electrophysiologic testing is an invasive procedure that assesses arrhythmias, there site of origin and treatment. It is generally reserved for patients with unidentified or resistant arrhythmias.

Choice E Drug therapy is appropriate for patients with intolerable symptoms or with a significant risk of progression to ventricular tachycardia or ventricular fibrillation. Beta-blockers are usually tried first because they are relatively safe drugs and have shown to reduce the risk of sudden death due to ventricular tachycardia or ventricular fibrillation in patients with heart disease. Anti-arrhythmic drugs can suppress ventricular premature beats, but may also increase the risk of a fatal arrhythmias. They should only be prescribed for carefully selected patients who have been evaluated for risk of developing serious arrhythmias.

An 12 year old male with hypoplastic left heart syndrome, now 11 years status post a surgical intracardiac total cavopulmonary connection by a

lateral tunnel (Fontan procedure) develops progressively worsening exercise tolerance and oxygen saturations. Further evaluation identifies dilation and

poor function of the patient's single ventricle with normal pulmonary vascular resistance. Despite maximal medical therapy, the patient's

symptoms persist, and his cardiac function does not improve.

Which of the following is the most appropriate statement regarding cardiac transplantation in this patient?

1) The patient's long term survival will be unchanged whether he receives a transplant or not 2) The patient's abnormal arterial and venous anatomy makes transplantation impossible 3) The patient's prior history of blood transfusions has likely led to alloimmunization that will rule out the possibility of transplantation 4) The patient will most likely develop severe coronary artery disease in the future if he receives a cardiac transplant 5) The patient's underlying disease will necessitate simultaneous heart and lung transplant

The correct answer is choice D.

For pediatric patients, cardiac transplantation remains the last option for chronic end-stage heart failure. Though often successful, cardiac transplantation is not without many serious complications. Rejection of the grafted organ is always possible, but if this is avoided, coronary vasculopathy is the major cause of patient and graft mortality. Coronary artery disease occurs at a much more rapid pace in patients with cardiac transplants who are receiving immunosuppressive medications; preventing this is the focus of intense research currently.

Page 5: Internal Medicine- Cardiology

In the right circumstance, long term survival (choice A) is greatly enhanced by cardiac transplantation. Generally, transplant should be considered when a patient's life expectancy due to their heart disease is only 1-2 years despite maximal medical and surgical therapy. Some 65% of patients are alive with a functioning graft at 10 years, meaning that a significant survival advantage is present.

Unusual arterial and venous anatomy (choice B) is common in patients with congenital heart disease, and almost never excludes the possibility of a transplant. Creative surgical techniques are often required, however.

Previous blood transfusions (choice C) can lead to alloimmunization, which can make obtaining a well matched transplant very difficult. (This is one reason that any patient in whom organ transplantation is a realistic future possibility should receive only blood products that have been reduced of lymphocytes, as it is the white blood cells that lead to HLA sensitization - red cells do not express these markers.) However, the presence of donor specific antibody does not rule out a transplant - it only requires different induction and immunosuppressive protocols. In fact, in infants, ABO incompatible donors have been used with very good success in some cases.

Simultaneous heart/lung transplantation (choice E) is required in patients whose cardiac disease is associated with pulmonary hypertension. (A good example is a patient with a left-to-right shunt who then develops reversal of the shunt or Eisenmenger's syndrome.) However, in this question stem you are specifically told that the pulmonary vascular resistance is normal

A 38-year-old man has had increasing dyspnea with peripheral edema, worsening for the past two years. On physical examination he has diffuse

crackles auscultated in both lungs. A chest radiograph shows that the heart nearly fills the entire thoracic chest. A chest CT scan demonstrates a 10 cm

mass involving the right atrium. On echocardiogram, the mass causes obstruction of the AV valve (a 'ball-valve' effect).

Which of the following neoplasms is this man most likely to have?

1) Rhabdomyosarcoma 2) Mesothelioma 3) Myxoma 4) Angiosarcoma 5) Papillary fibroelastoma

The correct answer is choice C.

Page 6: Internal Medicine- Cardiology

Atrial myxomas are the most common primary heart tumors. Because of nonspecific symptoms, early diagnosis may be a challenge. Left atrial myxoma may or may not produce characteristic findings on auscultation. Two-dimensional echocardiography is the diagnostic procedure of choice. Most atrial myxomas are benign and can be removed by surgical resection. Myxomas account for 40-50% of primary cardiac tumors. Approximately 90% are solitary and pedunculated, and 75-85% occur in the left atrial cavity. Up to 25% of cases are found in the right atrium. The mobility of the tumor depends upon the extent of attachment to the interatrial septum and the length of the stalk.

Symptoms are produced by mechanical interference with cardiac function or embolization. Being intravascular and friable, myxomas account for most cases of tumor embolism. Embolism occurs in about 30-40% of patients. The site of embolism is dependent upon the location (left or right atrium) and the presence or absence of an intracardiac shunt. Sudden death may occur in 15% patients with atrial myxoma. Death is typically caused by coronary or systemic embolization or by obstruction of blood flow at the mitral or tricuspid valve. Approximately 75% of sporadic myxomas occur in females.

Myxomas have been reported in patients aged 3-83 years. In about 20% of cases, myxoma may be asymptomatic and discovered as an incidental finding. Signs and symptoms of mitral stenosis, endocarditis, mitral regurgitation, and collagen vascular disease can simulate those of atrial myxoma. A high index of suspicion aids in the diagnosis. Symptoms range from nonspecific and constitutional to sudden cardiac death but also include:

Symptoms of left-sided heart failure

Symptoms of right-sided heart failure

Severe dizziness/syncope

Symptoms related to embolization; Systemic or pulmonary embolization

Constitutional symptoms that include fever, weight loss, arthralgias, and raynaud phenomenon are observed in 50% of patients. These symptoms

Page 7: Internal Medicine- Cardiology

may be related to overproduction of interleukin-6.

Hemoptysis due to pulmonary edema or infarction is observed in up to 15% of patients.

Chest pain is infrequent. If it occurs, it may be due to coronary embolization.

Jugular venous pressure may be elevated, and a prominent 'A' wave may be present.

A loud S1 is caused by a delay in mitral valve closure due to the prolapse of the tumor into the mitral valve orifice (mimicking mitral stenosis).

P2 may be delayed. Its intensity may be normal or increased, depending on the presence of pulmonary hypertension.

In many cases, an early diastolic sound, called a tumor plop, is heard. This sound is produced by the impact of the tumor against the endocardial wall or when its excursion is halted.

An S3 or S4 may be audible.

A diastolic atrial rumble may be heard if the tumor is obstructing the mitral valve.

If there is valve damage from the tumor, mitral regurgitation may cause a systolic murmur at the apex.

Page 8: Internal Medicine- Cardiology

A right atrial tumor may cause a diastolic rumble or holosystolic murmur due to tricuspid regurgitation.

General examination may reveal fever, cyanosis, digital clubbing, rash, or petechiae.

Patients with familial myxoma may have a variety of features known as the Carney complex, as follows: Spotty pigmentation such as lentigines (ie, flat brown discoloration of skin), pigmented nevi, or both

Endocrine hyperactivity such as Cushing syndrome

Multiple cerebral fusiform aneurysms may be seen in patients with Carney syndrome.

Other described syndromes associated with atrial myxomas include the following:

NAME syndrome features nevi, atrial myxoma, myxoid neurofibroma, and ephelides (ie, freckles [tanned macules found on the skin]).

LAMB syndrome features lentigines, atrial myxoma, and blue nevi.

Most cases of atrial myxoma are sporadic, and the exact etiology is unknown. Familial atrial myxomas have an autosomal dominant transmission.

Rhabdomyosarcoma (choice A) is a type of cancer, specifically a sarcoma (cancer of connective tissues), in which the cancer cells are thought to arise from skeletal muscle progenitors. It can also be found attached to muscle tissue, wrapped around intestines, or in any anatomic location. Most occur in

Page 9: Internal Medicine- Cardiology

areas naturally lacking in skeletal muscle, such as the head, neck, and genitourinary tract. Rhabdomyosarcoma is a relatively rare form of cancer. It is most common in children ages one to five, and is also found in teens aged 15 to 19, although this is more rare. This cancer is also an adult cancer but it is rare.

Malignancies involving mesothelial cells (choice B) that normally line the body cavities, including the pleura, peritoneum, pericardium, and testis, are known as malignant mesothelioma, which may be localized or diffuse. Most, but not all, cases of pleural malignant mesothelioma are associated with asbestos exposure.

Angiosarcoma (AS) (choice D) is an uncommon malignant neoplasms characterized by rapidly proliferating, extensively infiltrating anaplastic cells derived from blood vessels and lining irregular blood-filled spaces. Risk factors includes; Toxic exposure or radiation therapy, Other carcinogens (eg, bone wax, Dacron, metal bodies) Lymphedema. None of the risk factors is given in this scenario.

A papillary fibroelastoma (choice E) is a primary tumor of the heart that typically involves one of the valves of the heart. However, papillary fibroelastomas, considered generally rare, make up about 10 percent of all primary tumors of the heart. They are the third most common type of primary tumor of the heart, behind cardiac myxomas and cardiac lipomas.

A 78 year-old female with long-standing hypertension presents to the hospital with the acute onset of mid-sternal chest pressure and dyspnea. Findings

on physical examination include: Heart rate 110 per minute, regular; Blood pressure 200/100; Respiratory rate 24 breaths per minute; JVP 10 cm normal

upstrokes, Chest: Rales ½ of the way up bilaterally; Heart: Rapid S1 and S2 with a grade II/VI holosystolic murmur at the apex, radiating towards the

axilla; Extremities: no edema, cyanosis or clubbing.

The electrocardiogram reveals sinus tachycardia with ST elevation consistent with an acute inferior myocardial infarction.

The imaging study that would be most appropriate for evaluation of her systolic murmur would include which of the following:

1) Thallium stress test 2) Resting technetium scan 3) Transthoracic echocardiography 4) Right heart catheterization 5) Coronary angiography

Page 10: Internal Medicine- Cardiology

The correct answer is choice C

Ischemic mitral regurgitation is a complication of coronary artery disease. The holosystolic murmur radiating to axilla present in this 78-year old patient following myocardial infarction point to the development of mitral regurgitation consistent with infarction of papillary muscle.

2D echo (choice C) is considered the ―gold standard‖ for imaging abnormalities in valve morphology and motion. Doppler echocardiography is useful for diagnosis of valvular regurgitation while 2D echo is valuable for determining the etiology of valvular regurgitation as well as effect on dimension, shape and function.

Thallium stress test (choice A) is useful in determining the extent of blockage of coronary artery, causes of chest pain, level of exercise a patient can perform safely and effectiveness of cardiac procedures performed to improve coronary circulation.

Resting technetium scan (choice B) is useful in assessing the ventricular function. It gives an accurate estimate of ejection fraction.

Right heart catheterization (choice D) allows the measurement of pressure in right side of heart. Coronary angiography (choice E) is useful in detecting stenosis of coronary artery, collateral pathways perfusing the area of myocardium supplied by occluded artery, presence of congenital abnormalities of coronary artery, patency of coronary artery bypass grafts.

A woman, 44 year old, who has been treated for 2 months with propranolol to help lower her blood pressure comes to your office. She complains of experiencing nightmares which began shortly after starting the propranolol treatment. She is told to immediately stop taking the propranolol. Which of the following consequences will most probably occur as a result of an abrupt stopping of the propranolol treatment?

1) Continued drop in blood pressure 2) Relatively rapid asymptomatic return of the blood pressure to pre-treatment level 3) Slower asymptomatic return of BP to pre-treatment level 4) Rebound of the BP with symptoms and signs of sympathetic overactivity

Page 11: Internal Medicine- Cardiology

5) Overshoot of the BP above pretreatment levels

The correct answer is Choice D

Abrupt withdrawal of beta blockers often lead to a withdrawal syndrome, presumably due to increased sympathetic activity related to adrenergic receptor upregulation during the period of sympathetic blockade.

This syndrome is characterized by increased sympathetic activity, rebound hypertension, and possible accelerated angina or myocardial infarction.

The extent of sympathetic overactivity depends upon the relationship between the rate at which the antihypertensive wears off and the rate at which the receptors downregulate. Thus a hyperadrenergic state is most likely with short-acting drugs (such as propranolol) since receptor upregulation will persist after the antihypertensive effect has disappeared. In comparison, withdrawal syndromes are relatively unusual with longer-acting agents (such as nadolol).

In addition to a rise in BP, beta blocker withdrawal in patients with underlying coronary disease can lead to accelerated angina, myocardial infarction, or sudden death. This can occur even in patients who have no previous history of coronary symptoms.

To prevent withdrawal symptoms, these drugs should be slowly discontinued over a 6 to 10 day interval, cutting the dose by one-half every 2 to 3 days and switching to longer-acting drugs of the same class such as atenolol.

The development of pulmonary arterial hypertension in a patient with a large non-restrictive ventricular septal defect results in the attenuation of the

pansystolic murmur of VSD.

Which of the following statements is correct regarding the hemodynamics related to this phenomenon?

1) Development of right ventricular outflow obstruction due to infundibular pulmonic stenosis results in increase in right ventricular pressure attenuating the murmur

2) Large volume of blood passes through the lungs and reaches left atrium and attenuates the murmur of ventricular septal defect

Page 12: Internal Medicine- Cardiology

3) Shunting of blood through the defect occurs when right ventricle is contracting resulting in almost direct streaming of blood into the pulmonary artery

4) Large volume of blood passing through the normal pulmonary valve produces ejection systolic murmur at the pulmonary valve 5) Development of pulmonary arterial hypertension results in equalization of pressure across the defect

The correct answer is choice E

A large, nonrestrictive VSD is accompanied by a large left-to-right shunt, with subsequent development of pulmonary artery hypertension. Development of pulmonary arterial hypertension results in equalization of pressure across the defect (choice E). Paradoxically, infants with this type of defects present at a later age because the equalization of pressures across the defect attenuates the systolic murmur. A restrictive VSD is small and thus produces a significant pressure gradient between the left and right ventricle. It typically does not cause significant hemodynamic impairment and may eventually spontaneously close, but the turbulence of the high-pressure jet across the defect presents a high risk of endocarditis. Because of the high gradient, infants with a restrictive VSD demonstrate a loud murmur and often present at an early age. Development of right ventricular outflow obstruction due to infundibular pulmonic stenosis (choice A) results in increase in right ventricular pressure and the murmur of ventricular septal defect becomes an ejection systolic murmur. In this condition, pulmonary artery pressure in either normal or decreased. Large volume of blood passes through the lungs (choice B) reaches left atrium and passes through normal mitral valve resulting in delayed diastolic murmur at apex. Shunting of blood through the defect occurs when right ventricle is contracting (choice C) resulting in almost direct streaming of blood into the pulmonary artery and produces ejection systolic murmur at pulmonary valve. Shunting of blood through the defect results in large volume of blood coming to the right ventricle and passing through the normal pulmonary valve (choice D) produces ejection systolic murmur at the pulmonary valve.

Page 13: Internal Medicine- Cardiology

A 55-year-old male presents to emergency room with complains of dyspnoea at rest. He is having dyspnoea on exertion for last several months. General examination shows a pulse rate of 100 per minute, an elevated jugular venous pressure, a third heart sound, a pulsatile liver, ascites, and dependent edema. Chest X-ray is normal. Echocardiography shows mild left ventricular systolic dysfunction. The initial diagnostic workup should include all the following except:

1) CT scan of the chest 2) Treadmill exercise test 3) Fat pad biopsy 4) Iron studies 5) Tuberculin skin test

The correct answer is choice B The symptoms and findings on examination in this patient favour the diagnosis of right-sided heart failure. The differential diagnosis should include conditions in which there is circulatory congestion secondary to abnormal salt and water retention but in which there is no disturbance of cardiac structure or function (e.g., renal failure), noncardiac causes of pulmonary edema (e.g., acute respiratory distress syndrome), pulmonary vascular disease, restrictive cardiomyopathy, constrictive pericarditis, cor pulmonale, and any cause of longstanding left-sided heart failure. A CT or MRI of the chest (choice A) is useful for assessing pericardial calcifications or parenchymal lung disease which cannot be visualized on chest X-ray. Iron studies are useful for assessing hemochromatosis (choice D), and fat pad biopsy (choice C) is for evaluation of amyloidosis. Both these condition can cause restrictive cardiomyopathy. The tuberculin test is useful for determining the presence of prior infection with Mycobacterium tuberculosis, which is associated with the development of constrictive pericarditis (choice E). Treadmill test is not routinely recommended for patients with heart failure, but is useful for assessing the need for cardiac transplantation in patients with advanced HF (choice B).

A 70 year old african diabetic male presents to the hospital nearly 12 hours after the onset of severe substernal chest pressure. Physical examination

Page 14: Internal Medicine- Cardiology

reveals a heart rate of 110/minute, a blood pressure of 80/50, evidence of jugular venous distention, rales two thirds of the way up the lung fields and an S-3 gallop on cardiac exam. The EKG reveals sinus tachycardia at 110/minute with Q waves and ST segment elevations in the anterior leads. The most likely cause of this patient’s hypotension includes which of the following:

1) increased vagal tone 2) decreased stroke volume due to large anterior MI 3) vasodilation in compensation for the rapid heart rate 4) right ventricular infarction causing decreased preload 5) acute left ventricular free wall rupture

The correct answer is choice B. The scenario in this case shows that this patient has the typical scenario associated with an anterior Myocardial infarction. This icludes an ECG showing Q wave and ST segment elevations in the anterior leads, hypotensive state (BP 80/50), and Heart rate at 110 (tachycardia). Clinically, this is a picture of CARDIOGENIC SHOCK. Cardiogenic shock is characterized by a decreased pumping ability of the heart, or failure of the heart to pump effectively which results in a shock like state. It is most commonly occurs in association with, and as a direct result of, acute myocardial infarction (AMI). Similar to other shock states, cardiogenic shock is considered to be a clinical diagnosis characterized by decreased urine output, altered mentation, and hypotension. Other clinical characteristics include jugular venous distension, cardiac gallop, and pulmonary edema as in this case. The most recent prospective study of cardiogenic shock defines cardiogenic shock as sustained hypotension (systolic blood pressure BP less than 90 mm Hg lasting more than 30 min) with evidence of tissue hypoperfusion with adequate left ventricular (LV) filling pressure. Tissue hypoperfusion was defined as cold peripheries (extremities colder than core), oliguria (<30 mL/h), or both.

Cardiogenic shock is the leading cause of death in acute myocardial infarction (AMI).The overall in-hospital mortality rate is 57%. For persons older than 75 years, the mortality rate is 64.1%. For those younger than 75 years, the mortality rate is 39.5%. Outcomes significantly improve only when rapid revascularization can be achieved. The incidence of cardiogenic shock on arrival to the hospital has not changed significantly. Risk factors for the development of cardiogenic shock include pre-existing myocardial damage or disease (eg, diabetes, advanced age, previous AMI), AMI (eg, Q-wave, large or anterior wall AMIs), congenital heart disease, and dysrhythmia.

Increased vagal tone (choice A) causes bradycardia and not tachycardia as in this case. Pathologic causes include, but are not limited to, inferior wall myocardial infarction, toxic or environmental exposure, electrolyte disorders, infection, sleep apnea, drug effects, hypoglycemia, hypothyroidism, and

Page 15: Internal Medicine- Cardiology

increased intracranial pressure.

Normally vasodilation (choice C) lead to hypotension. However, as heart rate increases, it will leads to vasoconstriction and not vasodilation.

Right ventricular (RV) infarct (choice D), by itself, may lead to hypotension and shock because of reduced preload to the left ventricle. But should be considered in the setting of inferior wall MI.

In acute left ventricular free wall rupture (choice E), there will be no heart sound due to wall rupture, and the timing for acute left ventricular free wall rupture is usually 3 to 4 days post cardiac event.

A 53-year-old man with a past medical history of hypertension and coronary artery disease presents to the emergency department complaining of sharp

chest pain and shortness of breath that started about 1 hour ago. Four weeks ago, he was admitted to the hospital after having a myocardial infarction

(MI).

Cardiac enzymes, chest radiograph, and an electrocardiogram (ECG) are ordered. The ECG shows diffuse concave upward ST elevation, except aVR

and V1, which are depressed. T waves are not inverted.

Which medication should be given first to treat this condition?

1) Aspirin 650 mg 2) An thrombolytic agent such as tPA, intravenously 3) Sublingual Nitro spray 4) Propranolol 160 mg 5) Colchicine 1 mg 6) All of the above are true

The correct answer is Choice A.

Diffuse ST segment elevations without an anatomical distribution in the presence of fever, chest pain and dyspnea is consistent with a diagnosis of acute pericarditis. Given the patient’s recent myocardial infarction, the patient would be considered to have post-acute myocardial infarction syndrome until

Page 16: Internal Medicine- Cardiology

proven otherwise. Since it has been three weeks since the patient’s MI, this would fit the classic description of Dressler’s syndrome.

Dressler’s syndrome, also known as post-acute myocardial infarction syndrome, occurs between one to eight weeks after acute MI and includes symptoms of fever, chest pain (sharp, pressing, burning, or aching have been described), and pleuritic chest pain. post-acute myocardial infarction syndrome is relatively rare in the age of revascularization, but should still be recognized as a cause of acute pericarditis. Dressler’s syndrome is classically treated with 650-800 mg of aspirin (Choice A) or NSAIDs such as ibuprofen or indomethacin. Colchicine (Choice E) is also an effective treatment when aspirin or NSAIDs fail.

Not all ST segment elevation is acute MI. Thrombolytic therapy (Choice B), nitrates (Choice C), and beta-adrenergic receptor blockade (Choice D) are not standard treatment for post-MI pericarditis as they might be for an acute MI itself.

You are helping with school sports physicals and see a 13 year old boy who has had some trouble keeping up with his peers during gym class.

He has a cardiac murmur that you correctly diagnose as a ventricular septal defect based on which of the following auscultatory findings:

1) A systolic crescendo-decrescendo murmur heard best at the upper right sternal border with radiation to the carotids and augmented with transient exercise

2) A holosystolic systolic murmur at the apex 3) A holosytolic murmur at the mid left sternal border 4) A diastolic decrescendo murmur at the mid right sternal border 5) A continuous murmur through systole and diastole at the upper right sternal border

The correct answer is Choice C. In patients with ventricular septal defects (VSDs) and normal pulmonary artery pressures, murmurs are caused by turbulent blood flow following the pressure gradient from the left ventricle across the septal defect to the right ventricle during systole. Because blood begins flowing across the VSD prior to the opening of the aortic valve (i.e. during what would otherwise be the isovolumic phase of ventricular contraction), the resulting murmur is holosystolic.

Depending on the location of the defect within the ventricular septum, the associated murmur may be best heard at the left upper, mid-, or lower sternal border. When the right ventricle is chronically exposed to the higher left ventricular pressures due to large VSDs, reactive pulmonary hypertension may

Page 17: Internal Medicine- Cardiology

result. This, in turn, can cause Eisenmenger Syndrome, in which right ventricular pressures become chronically elevated, and the pressure gradient across the VSD becomes reduced or even reversed. Murmurs caused by VSDs typically lessen in intensity or disappear in the setting of Eisenmenger physiology, and patients may develop clinically significant right-to-left shunts.

Systolic murmurs at the right upper sternal border that radiate to the carotids (Choice A) are typically caused by aortic stenosis rather than VSDs. Murmurs best heard at the apex are typically caused by mitral valvular disease. A holosystolic murmur heard in this area (Choice B) is characteristic of mitral regurgitation.

Although the murmurs in patients with VSDs may occasionally have diastolic components caused by increased blood flow across the mitral valve, this murmur is typically best heard at the apex rather than at the right sternal border (Choices D and E).

A 68-year old smoker male with long-standing hypertension presents to the hospital with the acute onset of left-sided chest pain and dyspnea. Findings

on physical examination include: Heart rate: 120 per minute, regular; Blood Pressure: 180/90; Respirations: 24 per minute; Neck: 10 cm of JVP; normal

upstrokes; Chest: Crepitations at lung bases bilaterally; Extremities: no edema, cyanosis or clubbing. The electrocardiogram reveals sinus tachycardia

with ST elevation consistent with an acute anterior myocardial infarction.

All of the following statements about his acute myocardial infarction are true except:

1) The ―ischemic cascade‖ starting at the onset of her myocardial infarction includes diastolic dysfunction, systolic dysfunction, EKG changes and chest pain

2) The myocardial infarction proceeds in a ―wavefront‖ manner, proceeding from epicardium to endocardium 3) Systolic dysfunction caused by the infarction may lead to further ischemia by decreasing coronary perfusion pressure 4) Diastolic dysfunction caused by the infarction may lead to increased wall tension and decreased oxygen delivery to the myocardium 5) Sympathetic stimulation due to the pain of the infarction may increase myocardial oxygen demand

The correct answer is choice B

The events in ischemic cascade include decrease in left ventricular compliance, decreased myocardial contractility, increased left ventricular end-diastolic pressure, ST-segment changes and angina/chest pain (choice A).

Page 18: Internal Medicine- Cardiology

Infarction causes left ventricular systolic dysfunction. Systolic dysfunction leads to decreased systemic perfusion and hypotension leading to reduced coronary perfusion pressure and ischemia (choice C). Myocardial ischemia results in diastolic dysfunction which causes elevation in LV end diastolic pressure and myocardial wall stress. This impairs oxygen delivery to the myocardium (choice D).

Irreversible injury to ischemic myocardium develops in a transmural wavefront, occurs first at the subendocardial myocardium (choice B).

Myocardial infarction is accompanied by acute sympathetic stimulation. Pain also results in sympathetic stimulation (choice E) and leads to increase in myocardial oxygen demand.

A 46 year old woman with normal renal function is placed on an inappropriately high dose of thiazide diuretic for the complaint of mild ankle edema at the

end of the day. She returns 2 weeks later complaining of feeling weak, ―washed out‖, and dizzy if she stands up quickly. Examination confirms a

significant orthostatic fall in blood pressure. Electrolytes are drawn at this time.

Which of the following is the most likely electrolyte profile?

**Normal value of serum electrolytes

Sodium (Na+): 136-146 mEq/L

Potassium (K+): 3.5-5.0 mEq/L

Bicarbonate (HCO3-): 22-29 mEq/L

1) [Na+] 136 [K+] 5.4 [HCO3+] 32 2) [Na+] 154 [K+] 5.4 [HCO3+] 32 3) [Na+] 154 [K+] 2.7 [HCO3+] 21 4) [Na+] 136 [K+] 2.7 [HCO3+] 16 5) [Na+] 136 [K+] 2.7 [HCO3+] 32

The correct answer is choice E. The metabolic disturbance cause by thiazide diuretics include hypokalemic metabolic alkalosis and hyperuricemia. Hyponatremia is also an important

Page 19: Internal Medicine- Cardiology

adverse effect of thiazide diuretics. It is due to a combination of hypovolemia induced ADH elevation, reduction in the diluting capacity of the kidney, and increased thirst. Prescription of a thiazide diuretic without electrolyte monitoring can be associated with a high frequency of hyponatraemia and hypokalaemia in patients. Patients receiving higher doses of thiazides are particularly at risk of hypokalemia and elderly patients are at risk of hyponatraemia.

Page 20: Internal Medicine- Cardiology
Page 21: Internal Medicine- Cardiology

Review of Diuretics for Exam purposes

A. Diuretic agents affecting the proximal tubule: Carbonic anhydrase inhibitors (acetazolamide) prevent the normal breakdown of carbonic acid and, therefore, diminish bicarbonate reabsorption. Since the NaH antiporter is also involved in NaCl reabsorption, these agents also inhibit proximal tubule NaCl reabsorption. By diminishing sodium reabsorption, the osmotic gradient for water reabsorption is decreased. This leads to an increased delivery of NaHCO3, NaCl, and water from the proximal tubule to the remaining nephron. The thick ascending limb of Henle is well adapted to handling an increased load of NaCl, therefore, much of the increased delivery of NaCl is reabsorbed in the thick ascending limb of Henle. The rest of the nephron is not geared for bulk reabsorption of NaHCO3. The net result is moderate increase in sodium and bicarbonate in the urine along with an increase in urinary flow rate (water excretion). B. Osmotic diuretics: Substances which are filtered at the glomerulus but are not completely reabsorbed will act as osmotic diuretics. Examples are mannitol and glucose (when glucose has exceeded its maximum reabsorption capacity). Glucose is reabsorbed via a sodium coupled receptor which is saturable. Thus, when the filtered load of glucose exceeds the transport maximum of the proximal tubule for glucose, glucose becomes a non-reabsorbable osmotically active particle. This increase in osmotic activity attenuates the osmotic gradient for water reabsorption. Due to their effects on tubular fluid osmolality, osmotic diuretics also impair NaCl reabsorption in the proximal tubule and thick ascending limb of Henle. The net result is that osmotic diuretics are potent diuretics which lead to increased excretion of water and NaCl.

C. Loop diuretics (Furosemide): Loop diuretics must enter the tubular fluid to reach their site of action, which is the thick ascending limb of Henle. These diuretics block the luminal receptor which is responsible for the reabsorption of 1 sodium, 1 potassium, in conjunction with 2 chloride ions. Since the thick ascending limb is responsible for about 20% of the sodium chloride reabsorption, loop diuretics are extremely potent diuretics. They lead to increased sodium, potassium, chloride and water excretion.

D. Thiazide diuretics (Hydrochlorothiazide): These diuretics must enter the tubular fluid to reach their site of action in the early distal convoluted tubule. These diuretics block the luminal receptor which functions as an electroneutral sodium chloride transporter. The distal convoluted tubule is responsible for about 5% of the total sodium chloride

Page 22: Internal Medicine- Cardiology

reabsorption, thus, thiazide diuretics are of moderate potency. They lead to increased sodium, potassium, chloride, and water excretion.

E. The effect of solute excretion on urinary flow rate: Increased solute excretion (for example, increased NaCl excretion) leads to an increased urinary flow rate. The major mechanism for this is that the solute which is being excreted in increased amounts will be present in the tubular fluid in the collecting duct. Since the solutes are osmotically active, this will decrease the osmotic gradient for water reabsorption from the collecting duct to the interstitium. Thus, water excretion increases. This effect can have marked importance in patients who have abnormal urinary concentrating or diluting ability. In this situation changes in solute excretion will lead to marked changes in urinary flow rate.

F. Diuretics acting in the collecting duct: These agents act by either blocking the sodium channel in the luminal membrane (amiloride) or by acting as competitive antagonists for the cytoplasmic actions of aldosterone (spironolactone). The net result of either of these effects is to lead to a mild increase in sodium excretion. Since less sodium is being reabsorbed in the collecting duct, the lumen has a less negative potential, therefore, potassium secretion and hydrogen ion secretion are diminished. Thus, in contrast to diuretics which act before the collecting duct, these diuiretics lead to decreased potassium secretion.

G. Atrial natriuretic peptides (ANP): Small peptides hormones released primarily by the cardiac atria in response to increased atrial stretch, such as seen in volume expansion. ANP binds to receptors in the medullary collecting duct and lead to increased intracellular cyclic GMP. These changes lead to decreased sodium reabsorption. Increased solute excretion leads to increased water excretion. There is no change in the rate of potassium excretion.

H. Diuretic effects on potassium excretion: Diuretic agents that act at the proximal tubule (acetazolamide), loop of Henle (furosemide),distal convoluted tubule (thiazide), and osmotic diuretics lead to increased potassium excretion. Each of these diuretic agents increase delivery of sodium and filtrate to the cortical collecting duct, the major site of potassium secretion, leading to increased urinary potassium excretion. Diuretics that act at the cortical collecting duct decrease potassium excretion. ANP, acting at the medullary collecting duct has no effect on potassium excretion.

A 6 year old African-American male is brought to the clinic by his mother for fatigue and dark, cola-colored urine for the past three days. The child has not

Page 23: Internal Medicine- Cardiology

had fever, nausea or vomiting, or recent travel, though you had seen the patient last week for complaints of a rash

and started trimethoprim/sulfamethoxazole after diagnosing impetigo. Vital signs are temperature 36.8 C (98.2 F), pulse 98, blood pressure 106/52, and

respirations 14/min. Physical examination shows scleral icterus and a nontender abdomen without organomegaly. Laboratory evaluation shows:

WBC 8,600

Hgb 9.1 g/dL

Hct 27.3%

Platelets 212,000

MCV 88 fL

Sodium 144 mEq/L

Potassium 4.8 mEq/L

Chloride 101 mEq/L

Bicarbonate 26 mEq/L

BUN 14 mg/dL

Creatinine 0.8 mg/dL

Glucose 101 mg/dL

Total protein 6.8 g/dL

Albumin 4.2 g/dL

Total bilirubin 5.3 mg/dL

Direct bilirubin 0.3 mg/dL

AST (SGOT) 211 U/L

ALT (SGPT) 55 U/L

Alkaline phosphatase 101 U/L

LDH 410 U/L

Which of the following is the most appropriate next step in management of this patient?

1) Quantitative IgM for hepatitis A virus 2) Begin darbopoietin injections 3) Begin methylprednisolone 4) Discontinue trimethoprim/sulfamethoxazole 5) Transfusion of packed red blood cells

The correct answer is choice D.

Page 24: Internal Medicine- Cardiology

This is a case of glucose-6-phosphate dehydrogenase (G6PD) deficiency, an X-linked recessive disease that occurs commonly in males of Mediterranean or African descent.

G6PD is an enzyme in glycolysis that is responsible for the production of NADPH, which maintains glutathione in its reduced state and protects cells from oxidative stress. Since red blood cells are dependent on glycolysis and therefore G6PD to produce their NADPH and glutathione, deficiency of G6PD results in RBCs that are exquisitely susceptible to free oxygen radicals. Anything that increases oxidative stress results in hemolysis and an acute hemolytic anemia.

The most common culprits include: viral or bacterial infections, fava beans, nitrofurantoin, quinine, dapsone, and sulfonamides. Here, the culprit was sulfamethoxazole, so that drug should be discontinued (choice D).

The disease is typically self limiting, and treatment is supportive. The key diagnostic findings in this patient are the laboratory evidence of hemolysis, with an elevated LDH and AST, both of which indicate that RBCs are being lysed and are releasing their contents into the serum. The patient’s total and indirect (unconjugated) bilirubin are also elevated, which is unsurprising given the clinical appearance of jaundice.

Quantitative IgM for hepatitis A virus (choice A) is the best way to diagnose an acute hepatitis A infection, which can be a cause of jaundice. However, this patient’s jaundice is not due to his liver – it’s due to the fact that he is hemolyzing his red cells, as evidenced by the elevated LDH and low haptoglobin.

Darbopoietin injections (choice B) are the treatment for anemia due to chronic kidney disease, where the kidneys are not producing enough erythropoietin to stimulate the bone marrow to produce more RBCs. If you were to check a reticulocyte count on this patient, however, it would be elevated – his anemia is not from inadequate production of RBCs, it’s from hemolysis of RBCs, as shown by the increased LDH and decreased haptoglobin.

Corticosteroids like methylprednisolone (choice C) are used to treat autoimmune hemolytic anemias, which can present similarly to this patient. Only the clues about the patient’s race and the timing of the disease onset after starting TMP/SMZ steer you toward the diagnosis of G6PD deficiency. So why isn’t this the correct answer? Before starting a new drug, unless it is urgent that you do so, you should confirm the diagnosis. Here, that would involve a

Page 25: Internal Medicine- Cardiology

peripheral smear or Coombs’ testing to confirm a diagnosis of autoimmune hemolytic anemia. Your threshold for stopping a non-essential drug is usually lower, though, and if G6PD deficiency is on your differential diagnosis (and it should be) you ought to stop the potential offending drug while you try to tease out whether the patient’s symptoms are due to G6PD deficiency or autoimmune hemolysis. Keep in mind the order that you would do things - of the choices listed, stopping TMP/SMZ is the most appropriate NEXT step.

This patient does not need an immediate transfusion (choice E). Transfusions are reserved for patients with severe symptoms, worsening status, or fragile overall medical condition. This patient’s vital signs remain stable, and he is an otherwise healthy 6 year old, so although he should be monitored, he does not need a transfusion now.

A 75-year-old woman with a past medical history of hypertension is admitted with acute anterior septal wall infarction. She receives intravenous

thrombolytic therapy, and her chest pain resolves. Seventy-two hours later, her chest pain suddenly recurs and she develops respiratory distress and

becomes hypotensive. Physical examination reveals a new pansystolic murmur best heard at the left sternal border with thrill and S3 gallop. Pulmonary

capillary wedge tracings reveal some V wave prominence and oximetry reveals the following oxygen saturations: superior vena cava, 66%; inferior vena

cava, 70%; right atrial, 70%; right ventricular, 80%; and pulmonary arterial, 82%.

Which of the following is the most likely diagnosis based on these findings?

1) Cardiac rupture 2) Mitral regurgitation due to papillary muscle rupture 3) Ventricular aneurysm 4) Post-MI syndrome (Dressler syndrome) 5) Ventricular septal rupture

The correct answer is choice E. Complications of myocardial infarction (mostly Occurs Within the first 10 Days) include:

Recurrent MI

Page 26: Internal Medicine- Cardiology

Congestive heart failure

Arrhythmia

Pericarditis

Cardiogenic shock

Post Infarct Angina

Myocardial rupture, (LV free wall rupture, papillary muscle rupture and ventricular septal rupture)

Ventricular septal rupture (VSR) is a rare but a lethal complication occurs in a zone of necrotic myocardial tissue, and it usually occurs within the first 10-14 days. Clinical studies report an average time of 2.6, The age range of patients, from 44-81 years. Men are affected more commonly than women. In this question upon auscultation, a loud systolic murmur is heard, this is the most consistent physical finding of post infarction VSR. Prior to the development of the murmur, the patient's clinical course undergoes a sudden deterioration, with the development of congestive heart failure and, often, cardiogenic shock.

The typical harsh systolic murmur is audible over a large area, including the left sternal border and apical area. The murmur sometimes radiates to the left axilla, thereby mimicking mitral regurgitation. A thrill is palpable in approximately 50% of patients. An important diagnostic test for differentiating VSR from mitral valve insufficiency is catheterization of the right heart with a Swan-Ganz catheter. In the presence of a VSR, oxygen concentration between the right atrium and the pulmonary artery is stepped up. In addition, a pulmonary capillary wedge pressure tracing is beneficial for differentiating acute mitral regurgitation from VSR. Left- and right-sided pressure measurements help estimate the degree of biventricular failure and are useful in monitoring

Page 27: Internal Medicine- Cardiology

the response to perioperative therapy.

Cardiac rupture choice (A), and Mitral regurgitation due to papillary muscle rupture choice (B) are not correct answers, we need to differentiate which type of cardiac rupture cause this, as they mimick each other. The differential diagnosis for cardiac rupture are; VSR and mitral insufficiency secondary to papillary muscle rupture, papillary muscle dysfunction, or left ventricular wall rupture.

Ventricular aneurysm choice (C) is not a correct answer, A history of MI and third or fourth heart sounds are common findings with Ventricular aneurysms, but are commonly associated with post infarction VSR and contribute significantly to the hemodynamic compromise in these patients. The reported incidence rate of ventricular aneurysms ranges from 35-68%, whereas the incidence of ventricular aneurysms alone following MI without VSR is considerably less (12.4%).

Post-MI syndrome (Dressler syndrome) choice (D) is not a correct answer. Before the era of reperfusion, the incidence of post-MI syndrome ranged from 1-5% after AMI, but this rate has dramatically declined with the advent of thrombolysis and coronary angioplasty. Post-MI syndrome is considered to be an autoimmune process. Clinical features include fever, chest pain, and other signs and symptoms of pericarditis occurring 2-3 weeks after AMI, not 3 days as in this case

A patient who has an acute inferior myocardial infarction develops complete heart block within 24 hours of admission. The heart atrial rate is 80/min and

the ventricular rate is 45/min. The QRS duration is 80 msec. His blood pressure is 90/60 mm Hg. Respiratory rate is 16/min. There are no rales on lung

auscultation. The patient is confused.

Which one of the following statements is correct?

1) Complete heart block is the most common arrhythmia associated with inferior myocardial infarction 2) The incidence of complete heart block was significantly higher among patients with an anterior myocardial infarction than among those with

an inferior myocardial infarction 3) High degree atrioventricular block associated with inferior myocardial infarction is more often located below the AV node 4) The temporary transvenous pacing is an appropriate treatment in this situations 5) The condition is usually transient and associated with a low mortality rate

The correct answer is Choice E

Page 28: Internal Medicine- Cardiology

Conduction disturbances are commonly seen in inferior myocardial infarction, since the SA (sinoatrial) node, AV (atrioventicular) node and His bundle are primarily supplied by the right coronary artery. Sinus bradycardia, not complete heart block, is the most common arrhythmia associated with inferior myocardial infarction. It is present in up to 40 percent of patients in the first two hours, decreasing to 20 percent by the end of the first day. It is usually attributable to increased vagal tone in the first 24 hours after infarction (Choice A).

High (second or third) degree AV block associated with inferior wall myocardial infarction is located above the His bundle in 90 percent of patients (Choice C). For this reason, complete heart block often results in only a modest and usually transient bradycardia with junctional or escape rhythm rates above 40 beats per minute. It is usually seen within 24 hours after onset. It is associated with a narrow QRS complex, and develops in a progressive fashion from first to second to third degree block. It often results in an asymptomatic bradycardia and is usually transient, resolving within five to seven days and associated with a low mortality.

On the contrary, high degree AV block associated with anterior myocardial infarction is more often located below the AV node. It is usually symptomatic and has been associated with a mortality rate approaching 80 percent due in large part to greater loss of functioning myocardium (Choice E)

The incidence of complete heart block was significantly higher among patients with an inferior myocardial infarction than among those with an anterior myocardial infarction (9.4 versus 2.5 percent) (Choice B).

Bradyarrhythmias that occur early in the setting of an acute inferior myocardial infarction (within the first 24 hours) may respond to atropine, while those occurring later are often atropine-resistant. Treatment is not indicated in asymptomatic patients, but is useful if symptoms are present such as dizziness, syncope, or confusion from reduced cardiac output. Atropine is administered in 0.5 mg or 1 mg increments to a total of 3 mg. If the patient is hemodynamically stable and the escape rhythm has a narrow QRS and an adequate ventricular rate the placement of temporary pacemaker is not required but careful monitoring is mandatory in case of rapid deterioration (Choice D).

An 85-year old hypertensive male was referred to cardiologist for evaluation of his syncope. He is having a history of hypercholesterolemia for which he was receiving atorvastatin. On examination, his blood pressure is 150/80 mmHg, heart rate is 72 beats/min. On auscultation, he has a grade III/VI systolic ejection murmur which extends to S2 and radiates to carotids. S2 is barely audible. Femoral and abdominal aortic bruits are also present. Laboratory

Page 29: Internal Medicine- Cardiology

examination shows a creatinine of 0.9 mg/dL, low-density lipoprotein cholesterol of 75 mg/dL, high-density lipoprotein of 50 mg/dL. Echocardiographic findings are consistent with calcific aortic stenosis. All of the following are true regarding his condition except:

1) There is no medical therapy available for aortic valve stenosis 2) Use of statins slows down the progression of disease 3) Calcific changes primarily occur at the aortic side of the valve leaflets 4) Use of statin does not prevent structural valve deterioration of aortic valve bioprosthesis 5) High level of tissue plasminogen activator (tPA) in blood is associated with presence of aortic stenosis

The correct answer is choice B Calcific aortic valve disease (CAVD) is a slow but progressive pathological condition of the aortic valve characterized by dystrophic calcification of the valve leaflets. The murmur of aortic stenosis is classically described as a harsh, crescendo-decrescendo systolic murmur. It is most audible in the second right intercostal space and radiates to the carotid arteries. Patients often remain asymptomatic for years, during which no treatment is required. If a patient develops symptoms surgery is usually indicated. In addition, surgery is advocated when the ejection fraction falls below 50% or when severe calcification, rapid progression, or expected delays in surgery are present. There is no medical therapy available for aortic valve stenosis (choice A), surgery represents the only definitive therapy. Percutaneous balloon valvuloplasty has been used as a bridge to surgery and in patients with severe left-ventricular dysfunction or who cannot tolerate surgery. Several studies have tried to medically alter the progression of aortic stenosis by using statins without impressive results. It has been observed in trials that statins do not slow down the progression of aortic valve stenosis (choice B). The risk factors for atherosclerosis, such as age, male sex, smoking, diabetes mellitus, hypertension, chronic kidney disease, increased LDL, reduced HDL cholesterol, and elevated C-reactive protein are all risk factors for aortic valve calcification. For a long time, it was believed that CAVD occurs as a result of a simple degenerative process with passive accumulation of calcium in the cusps. Recently it has been suggested that CAVD is an active cellular process occurring inside valve leaflets. Mechanical stress on the aortic valve leads to valvular endothelial dysfunction/leakage, resulting in deposition of lipids and other compounds. This triggers inflammation, which in turn activates valvular myofibroblasts resulting in their osteoblastic transdifferentiation. These events lead to further changes involving extracellular matrix remodeling and neovascularization, ultimately leading to active

Page 30: Internal Medicine- Cardiology

calcification. These calcific changes primarily occur at the aortic side of the valve leaflets (choice C). Many different trials have been done to confirm the beneficial effects of statin therapy on structural valve deterioration of aortic valve bioprosthesis. Currently, studies and their results are discordant, and data is conflicting so, we can conclude that statin therapy does not prevent SVD of bioprosthetic valves in the aortic position (choice D). It has been reported that increased levels of tPA are present in patients with aortic valve stenosis. High levels of blood tPA are associated with the presence of aortic valve stenosis, thus representing a potential biomarker for CAVD (choice E).

Nitroglycerin administered sublingually may contribute to the relief of myocardial ischemic pain by each of the following mechanisms except: 1) Coronary vasodilation 2) Decreased diastolic perfusion time 3) Decreased arterial pressure 4) Decreased ventricular volume 5) Increases collateral flow

The correct answer is choice B

The given table summarizes the beneficial and deleterious effects of nitrates. Decreased diastolic perfusion time (choice B) is one of the deleterious effects of nitrates which result in decreased myocardial perfusion.

Table 1: Beneficial and Deleterious Effects of Nitrates in the Treatment of Angina

Page 31: Internal Medicine- Cardiology

A 57 year old female develops progressive dyspnea on exertion several weeks after a prolonged viral upper respiratory illness. She also notes fatigue,

orthopnea, and paroxysmal nocturnal dyspnea. An echocardiogram confirms a markedly dilated and hypocontractile left ventricle, with a left ventricular

ejection fraction of 20%, and she is diagnosed with a dilated cardiomyopathy. As part of her treatment, the patient receives an ACE inhibitor.

All of the following statements concerning ACE inhibitors are correct, except:

1) They may produce a persistent dry cough as a side effect. 2) They produce an increase in circulating aldosterone levels. 3) They decrease vascular resistance. 4) They inhibit the formation of Angiotensin II

Page 32: Internal Medicine- Cardiology

5) They should not safe in pregnant women

The correct answer is Choice B. ACE inhibitors are among the most commonly prescribed cardiovascular medications. Angiotensin converting enzyme (ACE) facilitates several biochemical reactions including the conversion of Angiotensin I to Angiotensin II and the conversion of bradykinin to inactive peptides. Angiotensin II stimulates the adrenal glands to release aldosterone. By decreasing Angiotensin II production, ACE inhibitors decrease (rather than increase) aldosterone levels. ACE inhibitors cause a dry cough (Choice A) in 5 to 10 percent of patients. Other side effects include angioedema, hyperkalemia, and hypotension, and ACE inhibitors can also precipitate acute renal failure in patients with bilateral renal artery stenosis. Angiotensin II causes vasoconstriction with a resultant increase in vascular resistance. ACE inhibitors decrease the expression of Angiotensin II and, as a result, decrease peripheral resistance (Choices C and D).

Women who take ACE inhibitors during the second or third trimester of pregnancy experience increased rates of obstetrical complications and birth defects including pulmonary hypoplasia, intrauterine growth retardation, oligohydramnios, renal dysgenesis, and neonatal death. ACE inhibitors are therefore contraindicated during pregnancy (Choice E).

Page 33: Internal Medicine- Cardiology

Beta blockers are proven to be an essential component of the management of patients with acute myocardial infarction.

The beneficial effects of beta blockers in this setting include all of the following EXCEPT:

1) Decreased oxygen demand 2) Decreased risk of ventricular fibrillation 3) Decreased automaticity 4) Decreased coronary diastolic perfusion

Page 34: Internal Medicine- Cardiology

5) Reduction in remodeling of the left ventricle

The correct answer is Choice D

For a broad range of patients with both ST elevation and non-ST elevation myocardial infarction, including those with and without revascularization, the administration of a beta blocker reduce short-term complications, improve long-term survival and reduce morbidity and mortality due to the disease.

The beneficial properties of beta blockers in case of myocardial damage are the following:

Decreased oxygen demand due to the reductions in heart rate, blood pressure, and contractility, and the consequent relief of ischemic chest pain (Choice A)

Decreased risk of ventricular fibrillation due to an increase in the ventricular fibrillation threshold and consistently a significant reduction in risk of sudden cardiac death (Choice B)

Decreased automaticity, increased electrophysiologic threshold for activation, and slowing of conduction (Choice C)

Bradycardia prolongs diastole and therefore improves (not decrease) coronary diastolic perfusion (Choice D)

Reduction in remodeling and improvement in left ventricular hemodynamic function. The effect depends upon infarct size and the timing of treatment (Choice E)

Page 35: Internal Medicine- Cardiology

Improved left ventricular diastolic function with a less restrictive filling pattern.

Slowing of progression of coronary atherosclerosis in patients with and without MI, as demonstrated by intravascular ultrasound evaluation of atheroma volume

Due to all above the current guidelines recommend initiation of oral beta blockers within the first 24 hours in all patients with acute myocardial infarction in the absence of contraindications. Patients who do not receive a beta blocker during the first 24 hours because of early contraindications should be reevaluated for beta blocker candidacy for subsequent therapy. If tolerated the treatment with oral beta blockers should be continued indefinitely in all patients who have sustained an acute MI.

A twenty-eight year old white male is brought to the emergency room after a syncopal episode while playing basketball. The patient’s maternal

grandfather and maternal aunt died of ―heart attacks‖ at the age of 42 and 37 respectively. Physical examination reveals a bifid carotid pulse and a harsh

crescendo-decrescendo systolic murmur which accentuates with standing up or Valsalva maneuver and radiates to the left axilla. An ECG is remarkable

for left axis deviation and a left ventricular strain pattern.

Treatment of this patient might include each of the following, except:

1) Surgical resection of a portion of the upper interventricular septum 2) Alcohol septal ablation of a localized area of the upper septum 3) Medical therapy with a calcium channel blocker 4) Medical therapy with digoxin 5) Medical therapy with a beta blocker

The correct answer is Choice D. The patient’s presentation is strongly suggestive of hypertrophic cardiomyopathy (HCM). Familial HCM follows an autosomal dominant pattern of inheritance with incomplete penetrance. Mortality from HCM may result from malignant arrhythmias causing sudden cardiac death. Because this often occurs outside of a healthcare setting, family members may never learn the actual underlying cause of death. It is therefore important to ask about unusual or unexplained death (particularly at a young age) when heritable causes of arrhythmia are in the differential.

Page 36: Internal Medicine- Cardiology

Patients with HCM may present with dyspnea, angina, and presyncope/syncope on exertion. The murmur and bifid arterial pulse described in this case are both associated with HCM, and EKG may show a variety of findings suggestive of LVH.

Patients with HCM are unlikely to benefit from treatment with digoxin (Choice D).

Pharmacologic treatment modalities for treating HCM include beta-blockers (Choice E) and calcium channel antagonists (Choice C). Patients who fail medical management may benefit from ventricular septal myotomy/myomectomy (Choice A) or septal ablation with alcohol (Choice B).

A 55-year-old hypertensive man develops sudden onset of excruciating pain beginning in the anterior chest, and then radiating to the back. Over the next

2 hours, the pain moves downward toward the abdomen. Investigations showed a neutrophil leukocytosis and the hemoglobin level, urea, electrolytes

and liver function tests were normal. An ECG only showed sinus tachycardia and no other acute changes and a chest-X-ray shows widening of the

mediastinum.

Which of the following is the most likely diagnosis?

1) Aortic dissection 2) Syphilitic aneurysm 3) Aortic valve stenosis 4) Atherosclerotic aneurysm (AAA) 5) Acute myocardial infarction (AMI)

The correct answer is choice A. Aortic dissection is defined as separation of the layers within the aortic wall. Tears in the intimal layer result in the propagation of dissection (proximally or distally) secondary to blood entering the intima-media space, creating a false lumen and leading to a hematoma that propagates longitudinally. This occurs most commonly 2° to hypertension. Most classic aortic dissections begin at 1 of 3 distinct anatomic locations, including (1) the aortic arch, (2) approximately 2.2 cm above the aortic root, or (3) distal to the left subclavian artery.

Page 37: Internal Medicine- Cardiology

This condition most often occurs between 40–60 years of age. Patients with Marfan syndrome present earlier, usually in the third and fourth decades of life. The incidence of aortic dissection is estimated to be 5-30 cases per 1 million people per year. Aortic dissection occurs once per 10,000 patients admitted to the hospital; approximately 2,000 new cases are reported each year in the United States. Aortic dissection is more common in males than in females, with a male-to-female ratio of 2:1. Aortic Dissection may be caused by:

Hypertension (70% have elevated blood pressure)

Iatrogenic causes: cardiac surgery, including aortic and mitral valve replacements, coronary artery bypass graft surgery, or percutaneous catheter placement

Aortic atherosclerosis

Factors include cystic medial necrosis and aortic medial disease

Congenital aortic valve anomalies including unicommissural or bicuspid aortic valves or aortic coarctation

Marfan syndrome

Advanced age

Pregnancy

Page 38: Internal Medicine- Cardiology

Ehlers-Danlos syndrome

Syphilitic aortitis

Deceleration injury possibly with related chest trauma

Aortic arch hypoplasia

Cocaine use

There are two systems of classification for aortic dissection:

DeBakey system: Classifies dissections as involving both the ascending and descending aorta (type I), confined to the ascending aorta (type II), or confined to the descending aorta (type III).

Stanford system: Classifies dissection of the ascending aorta as type A and all others as type B.

Patients with acute aortic dissection present with the sudden onset of severe and tearing chest pain.The pain is usually localized to the front or back of the chest, often the interscapular region, and typically migrates with propagation of the dissection as in this case. Anterior chest pain is a manifestation of

Page 39: Internal Medicine- Cardiology

ascending aortic dissection. Neck or jaw pain is a manifestation of aortic arch dissection. Interscapular tearing or ripping pain is a manifestation of descending aortic dissection. Neurologic deficits are a presenting sign in up to 20% of cases. Syncope is part of the early course of aortic dissection in approximately 5% of patients and may be the result of increased vagal tone, hypovolemia, or dysrhythmia.

Cardiovascular manifestations are secondary to congestive heart failure as a result of severe aortic regurgitation. Clinically this presents with dyspnea, orthopnea, bibasilar crackles, or elevated jugular venous pressure. Signs of aortic regurgitation include bounding pulses, wide pulse pressure, and diastolic murmurs. Hypertension may result from a catecholamine surge or underlying essential hypertension. Hypotension is an ominous finding and may be the result of excessive vagal tone, cardiac tamponade, or hypovolemia from rupture of the dissection.

Other cardiovascular manifestations include findings suggestive of cardiac tamponade. Superior vena cava syndrome can result from compression of the superior vena cava from a large, distorted aorta. Wide pulse pressure and pulse deficit or asymmetry of peripheral pulses is reported. Patients with right coronary artery ostial dissection may present with acute myocardial infarction, commonly inferior myocardial infarction. Pericardial friction rub may occur secondary to pericarditis. Respiratory symptoms can include dyspnea and hemoptysis if dissection ruptures into the pleura or if tracheal or bronchial obstruction has occurred. GI symptoms include dysphagia, flank pain, and/or abdominal pain. Dysphagia may occur from compression of the esophagus. Flank pain may be present if the renal artery is involved. Abdominal pain may be present if the dissection involves the abdominal aorta.

ECG testing is usually normal, CXR (shows widening of the mediastinum, cardiomegaly, or new left pleural effusion). CT angiography is the gold standard of imaging. TEE can provide details of the thoracic aorta, the proximal coronary arteries, the origins of arch vessels, the presence of a pericardial effusion, and aortic valve integrity.

Syphilitic aneurysm (choice B) is associated with the tertiary stage of syphilis infection. If a patient has untreated syphilis, the infection may spread to the part of the aorta nearest the heart, resulting in a thoracic aortic aneurysm 15 to 30 years after the first signs of syphilis. In this case, no sexual history has been given to suspect this.

Most commonly, aortic stenosis (choice C) is due to age-related progressive calcification of the normal trileaflet valve. Aortic stenosis does not always cause symptoms immediately, even though the valve can be tight. Characteristic signs include a systolic heart murmur, fatigue, weakness, shortness of breath with exercise or at night, angina (chest pain) and fainting upon exertion (syncope).

Page 40: Internal Medicine- Cardiology

Most intact aortic aneurysms (choice D) do not produce symptoms. As they enlarge, symptoms such as abdominal pain and back pain may develop. Compression of nerve roots may cause leg pain or numbness.

The pain of aortic dissection is typically distinguished from the pain of acute myocardial infarction (choice E) by its abrupt onset, although the presentations of the two conditions overlap to some degree and are easily confused. Aortic dissection can be presumed in patients with symptoms and signs suggestive of myocardial infarction but without classic ECG findings.

A 25-year-old previously healthy woman dies suddenly and unexpectedly. She had complained only of a slight headache for 3 days before her demise. At

autopsy, the medical examiner finds an enlarged, dilated 410 gm heart with only minimal coronary atherosclerosis and normal cardiac valves.

Microscopically, the myocardium on both the right and the left ventricles demonstrates infiltration by small lymphocytes, with focal myocyte necrosis.

Which of the following infectious agents is most likely to have caused these findings?

1) Coxsackie B virus 2) Candida albicans 3) Aspergillus fumigatus 4) Streptococcus, viridans group 5) Staphylococcus aureus

The correct answer is choice A

Coxsackie viruses belong to the family Picornaviridae and the genus Enterovirus, which also includes poliovirus and echovirus. Coxsackieviruses are nonenveloped viruses with linear single-stranded RNA. Coxsackieviruses are divided into group A and group B viruses. In general, group A coxsackieviruses tend to infect the skin and mucous membranes, causing herpangina, acute hemorrhagic conjunctivitis (AHC), and hand-foot-and-mouth (HFM) disease. Group B coxsackieviruses tend to infect the heart, pleura, pancreas, and liver, causing pleurodynia, myocarditis, pericarditis, and hepatitis.

Coxsackie viruses are transmitted primarily via the fecal-oral route and respiratory aerosols. The viruses initially replicate in the upper respiratory then dissemination to target organs occurs following a secondary viremia.

Page 41: Internal Medicine- Cardiology

In heart it causes myopericarditis. Most patients are presented with dyspnea, chest pain, fever, malaise up to heart failure and death.

Histopathological findings include focal zones of myocyte necrosis involving polymorphonuclear leukocytes and in late phase there would be mononuclear cells within healing foci of myocyte necrosis as well as a distinctive diffuse interstitial pattern of myocarditis.

Complications include pericardial effusion, arrhythmia, heart block, valvular dysfunction, and dilated cardiomyopathy.

Candida albicans (choice B) can cause myopericarditis which occurs as diffuse abscesses scattered throughout the myocardium surrounded by normal cardiac tissue.

Aspergillus fumigatus (choice C) cause abscesses in the myocardium. Histopathological findings include characteristic septate hyphae, branching at acute angles, and acute inflammatory infiltrate and tissue necrosis with occasional granulomata and blood vessel invasion.

Streptococcus, viridans group (choice D) is the most common cause of subacute infective endocarditis. It is either predisposed by or causes valvular damage (cardiac valves are normal in this case)

Staphylococcus aureus (choice E) is the most common cause of infective endocarditis which is almost always associated with valvular damage. The hallmark of staphylococcal infection is the abscess, which consists of a fibrin wall surrounded by inflamed tissues enclosing a central core of pus containing organisms and leukocytes.

Which of the following statements is incorrect about hypertrophic cardiomyopathy? 1) Hypertrophic cardiomyopathy is primarily a disease of cardiac myocyte characterized by hypertrophy of cardiac myocyte 2) HCM is the most common cause of sudden cardiac death in young athletes 3) Beta-blockers are effective in preventing and reversal of cardiac hypertrophy 4) The causal genes for hypertrophic cardiomyopathy encode for sarcomeric protein

Page 42: Internal Medicine- Cardiology

5) The two most common genes are MYH7 and MYBPC3

The correct answer is choice C Hypertrophic cardiomyopathy (HCM) is a primarily a disease of cardiac myocytes, characterized mainly by cardiac and myocyte hypertrophy (choice A). It is diagnosed clinically by the presence of cardiac hypertrophy in the absence of altered loading conditions that may account for the problem. Typically, the left ventricle is not dilated and the left ventricular ejection fraction is increased. HCM is the prototypic and genetic form of pathologic cardiac hypertrophy. Hypertrophic cardiomyopathy is an archetypical single-gene disorder. More than a dozen causal genes and several hundred mutations have been identified. The known causal genes encode sarcomeric proteins (choice D). Therefore, HCM is commonly recognized as a disease of sarcomeric proteins. The 2 most common genes—each accounting for approximately 25% of cases—are MYH7 and MYBPC3 (choice E), which encode the β-myosin heavy chain and myosin-binding protein C, respectively. TNNT2, TNNI3, TPM1, and ACTC1 collectively account for about 10% of cases; other causal genes, such as the Z disk proteins MYOZ2 and TCAP, encoding myozenin 2 and telethonin, respectively, are less common. In young athletes, HCM is the most common cause of sudden cardiac death (SCD) and is responsible for almost half of all cases (choice B). Sudden cardiac death is often the first manifestation of HCM in apparently healthy, young individuals. The fundamental question in HCM is whether cardiac hypertrophy and fibrosis can be reversed or prevented once it has been established. None of the current pharmacologic agents have yet prevented, or reversed the cardiac hypertrophy or altered the prognosis of HCM. ?-Blockers remains the mainstay of therapy and are effective for symptomatic relief but not for the reversal of cardiac phenotype. The clinical usefulness of calcium channel blockers is limited by the risk of hypotension and syncope. Their effectiveness in the reversal of cardiac phenotype is yet to be established.

A 60-year-old ex-smoker man presents to the hospital with complains of cough, dyspnea on exertion and abdominal distention. His physical tolerance

decreased significantly during last several months but he sleeps well at night.

On physical examination the patient has elevated jugular venous pressure, hepatomegaly, ascites and peripheral edema. His blood pressure is 130/80

mm Hg, heart rate is 80/min, regular. The respiratory rate is 16/min at rest. Auscultation of the heart reveals third heart sound and no murmurs. There is

Page 43: Internal Medicine- Cardiology

prolonged expiration and wheezes on forced exhalation on lung auscultation. Chest X-ray shows hyperventilation, no pleural effusions, no cardiomegaly.

ECG is normal.

What is the most cause of the patient’s dyspnea?

1) Ischemic heart disease 2) Cor pulmonale 3) Dilated cardiomyopathy 4) Valvular dysfunction 5) Cardiac tamponade

The correct answer is Choice B Heart failure is a complex clinical syndrome that can result from any structural or functional cardiac disorder that impairs the ability of the ventricle to fill with or eject blood. Symptoms of heart failure include those due to excess fluid accumulation (dyspnea, orthopnea, edema, pain from hepatic congestion, and abdominal distention from ascites) and those due to a reduction in cardiac output (fatigue, weakness) that is most pronounced with exertion. Cor pulmonale is the alteration of right ventricular structure or function that is due to pulmonary hypertension caused by diseases affecting the lung or its vasculature. The most frequent cause of cor pu lmonale is chronic obstructive pulmonary disease (COPD) due to chronic bronchitis or emphysema. COPD and heart failure may be difficult to distinguish in some patients. Because of the high prevalence of these disorders, their similar presentations, and their frequent coexistence, it is reasonable to consider both diagnoses if the patient presents with deterioration in respiratory status. Our patient can be diagnosed with COPD due to presence of previous history of smoking, chronic cough, dyspnea with exertion, prolonged expiration and wheezes on lung auscultation and hyperventilation on chest x-ray (Choice B). For confirmation of the diagnosis the patient should undergo pulmonary function testing (PFTs) to determine the severity of the airflow obstruction. The absence of left heart failure symptoms (orthopnea, pulmonary rales, pleural effusion) and normal cardiac silhouette on chest x-ray make the other diagnoses (cardiac tamponade, dilated cardiomyopathy or ischemic heart disease) less likely (Choices A, C, E). There is no murmur on cardiac auscultation to suspect valvular disease (Choice D). The most useful test for exclusion the above conditions is

Page 44: Internal Medicine- Cardiology

echocardiography.

The patient is diagnosed with ischemic cardiomyopathy. The echocardiography shows the left ventricular ejection fraction (LVEF) of 40 percent. Currently

his condition is stable and there is no evidence of fluid retention. His blood pressure is 110/60mm Hg, heart rate is 80/min, respiratory rate is 18/min. He

is taking enalapril 10 mg twice daily.

What regime of beta adrenergic blocker therapy should be prescribed to the patient?

1) Beta blockers should not be prescribed to the patient because they impair left ventricular contractility further 2) Beta blockers should not be prescribed to this patient because they decrease long-term survival of such patients 3) Atenolol 25 mg once daily, then double the dose in two weeks if tolerated 4) Bisoprolol 1.25 mg once daily; then double the dose in two weeks if tolerated 5) Carvedilol 50 mg twice daily if no contraindications

The correct answer is Choice D Randomized trials have shown that blockade of beta adrenergic receptors leads to symptomatic improvement, reduced hospitalization and enhanced survival in many patients with heart failure (HF) and systolic dysfunction. Therefore, beta blockers are an important component of standard HF therapy in patients with current or prior symptoms of HF and depressed left ventricular (LV) systolic function (Choice A, B) Several major trials confirmed that use of certain beta blockers, such as carvedilol, extended release metoprolol succinate, and bisoprolol, reduces hospitalizations for HF and improves survival in patients with HF due to systolic dysfunction. Although retrospective data suggest a possible beneficial effect in HF of some beta blockers without proven benefit in randomized controlled trials (eg, atenolol) these studies do not provide a sufficient basis for recommendation of their use for HF treatment (Choice C). Prior to initiation of therapy, the patient should have no or minimal evidence of fluid retention. ACE inhibitor therapy should generally be instituted before beta blocker therapy is initiated. Beta blocker therapy provides incremental benefit in patients with HF treated with an ACEI inhibitor or an ARB or an aldosterone antagonist. Beta blocker therapy should be begun at very low doses and the dose doubled at regular intervals (for example, every two to three weeks) until the target

Page 45: Internal Medicine- Cardiology

dose is reached or symptoms (worsening HF) become limiting or symptomatic hypotension or excessive bradycardia (rate <50 beats/min) develops. Every effort should be made to achieve the target doses through individualized dose titration. When target doses are not tolerated, even low doses appear to be of benefit, although they are not optimal (Choice E).

On screening test your patient , 45 years old, is found to have serum triglyceride level of 200 mg/dL (2.3 mmol/L). All other lipid profile measures are

normal. His body mass index is 27kg/m2. He does not have any complains and tolerates physical activity well. His blood pressure is 130/70 mm Hg, the

heart rate is 66/min, regular, the respiratory rate is 12/min. He does not smoking. He does not take any medication.

What is the next step in managing the patient?

1) Repeat the screening test in five years 2) Order fasting blood glucose, serum creatine, thyroid function test 3) Prescribe atorvastatin 4) Prescribe nicotinic acid 5) Prescribe gemfibrozil

The correct answer is Choice B

Hypercholesterolemia is one of the major modifiable risk factors for coronary heart disease that is why current guidelines recommend the screening for lipid abnormalities to be performed for all adults at least once every five years. Screening should begin with a standard lipid profile that includes total cholesterol, low density lipoprotein (LDL)-cholesterol, high-density lipoprotein (HDL)-cholesterol and triglycerides. Individuals without known CHD and who have a desirable serum LDL cholesterol concentration can be rescreened in five years (Choice A).

Hypertriglyceridemia is defined as a serum triglyceride concentration above 200 mg/dL (2.3 mmol/L). Most often it is induced or exacerbated by secondary, potentially correctable disorders such as obesity, diabetes mellitus, nephrotic syndrome, hypothyroidism. So in case of discovered hypertriglycemia laboratory studies should always be ordered to exclude secondary disorders of lipid metabolism (Choice B)

Remember that a number of medications can also lead to the increased triglycerides level. These are beta blockers (particularly nonselective agents), glucocorticoids, cyclosporine, estrogens. Thus, medication review should always be performed in case of hypertriglyceridemia too. The described patient doesn’t take any medication currently.

Page 46: Internal Medicine- Cardiology

The first-line therapy for this patient should be nonpharmacologic interventions such as weight loss (his body mass index is 27kg/m2), aerobic exercise, avoidance of concentrated sugars and increase omega-3 fatty acid intake.

If a patient is found to have high triglycerides along with high LDL cholesterol level or high risk of ischemic heart disease, the priority of the therapy becomes firstly to decrease non-HDL cholesterol to the goal level according the main guidelines. It may require intensifying therapy with an LDL cholesterol lowering drugs (statins).The more potent statins (eg, atorvastatin or rosuvastatin) will control the LDL-C and, if the LDL-C is proportionately more elevated than serum triglycerides, the hypertriglyceridemia as well. If additional triglyceride lowering is required, either a fibrate or nicotinic acid can be added. Our patient has normal lipid profile except triglycerides and his coronary risk is not high so there is no indication for pharmacologic therapy now (Choices C, D, E).

A 17-year-old girl is brought to your office for assessment because she remains short in stature for her age. She has not shown any changes as well that

are consistent with puberty.

On physical examination, her vital signs include the following:

Body temperature: 37 degrees Celcius

Respiratory rate: 18/minute

Pulse rate: 75/minute

Blood pressure:165/85 mm Hg

She has a continuous murmur heard over both the front of the chest as well as her back. Her lower extremities are cool with poor capillary filling. She has

a webbed neck on further examination. A chest radiograph reveals a prominent left heart border, no edema or effusions, and significant rib notching.

Which of the following cardiovascular abnormalities is she most likely to have?

1) Shortening and thickening of chordae tendineae of the mitral valve 2) Coarctation of the aorta past the ductus arteriosus 3) Supravalvular narrowing in the aortic root 4) Lack of development of the spiral septum and partial absence of conus musculature 5) Single large atrioventricular valve

The correct answer is choice B.

Page 47: Internal Medicine- Cardiology

Coarctation of the aorta may be defined as a constricted aortic segment. In 98% of cases, it occurs immediately beneath the origin of the left subclavian artery at the site ductus arteriosus (juxtaductal coarctation). However, rarely, it may occur distal to the ductus arteriosus as in this case due to the prolonged survival (the age of the patient at the firs presentation is 17).

In 15-20% of cases it is associated with Turner syndrome (45,X) which is characterized by the following features: short stature, webbed neck and delayed puberty (all seen in this case).

This narrowing of the aorta imposes significant afterload on the left ventricle leading to compensatory ventricular hypertrophy (prominent left cardiac border on x-ray). Hypertension develops mainly (blood pressure is 165/85) due to:

Mechanical obstruction- this elevates the blood pressure proximal to coarctation.

Activation of the renin-angiotensin system secondary to reduction of renal blood flow. This explains explain the increased peripheral vascular resistance distal to the site of obstruction.

On auscultation, there would be a continuous and/or late systolic murmur that is best heard posteriorly over the thoracic spine due turbulent blood flow through the stenosed segment. It doesn't affect the right side of the heart and the pulmonary circulation (so there is no tachypnea or pulmonary edema). Rib notching in the x-ray is due to dilatation collateral vessels that connect arteries from the upper part of the body to the vessels below the level of coarctation.

Usual complications of this case includes recurrent coarctation and persistent hypertension after surgical repair, aortic aneurysm and congestive heart failure.

Page 48: Internal Medicine- Cardiology

Shortening and thickening of chordae tendineae (choice A) is commonly a feature of mitral stenosis in which there would be pulmonary congestion, tachypnea and a diastolic murmur with presystolic accentuation best heard at apex rather than a systolic murmur.

Supravalvular narrowing of the root of aorta (choice c) would be presented early in life by triad of aortic stenosis includes angina pectoris, syncope, and heart failure feature including exertional dyspnea.

The attached photo shows aortic coarctation visualized by aortic angiography.

Choice D is also false. The spiral septum normally separates the aorta and pulmonary artery, so its absence results in a communication between the two specially when the conus, a conical pouch from which the pulmonary artery arises, is also absent. Usually it is presented with presents with typical signs and symptoms of congestive heart failure including pulmonary congestion and dyspnea.

A single atrioventricular valve (choice E) causes regurgitation of blood into the right and/or left atrium with ventricular contraction. The overall problems are similar to those of VSD including pulmonary congestion. Usually presents by 6-8 weeks of age with tachypnea, repeated respiratory infections, poor feeding, and failure to thrive.

Page 49: Internal Medicine- Cardiology

A 30 year old woman presents with the chief complaint of shortness of breath with minimal activity. In retrospect, she feels she has been dyspneic for at

least 1 year but has now progressed to the point where she has difficulty climbing stairs or walking short distances. She denies fever, cough, or chest

pain.

On physical examination, the patient has jugular venous distention (JVD) and a palpable right ventricular lift. On heart auscultation, there is a loud S2 and

a systolic murmur that increases with inspiration. Lungs are clear. There is no clubbing.

Which of the following is the most likely diagnosis?

1) Congestive Heart Failure 2) Coronary heart disease 3) Idiopathic pulmonary fibrosis 4) Primary pulmonary hypertension 5) Systemic lupus erythematosus

The correct answer is Choice D.

Primary pulmonary hypertension (Choice D) is a rare disease that occurs most often in women of childbearing age and is associated with increased pulmonary artery pressures with no identifiable cause. Primary pulmonary hypertension causes dyspnea on exertion (without overt chest pain), jugular venous pulsations, tricuspid regurgitation, and a right ventricular lift (heave). There is a strong genetic component to this disease, which makes careful family history important along with a discussion of possible toxic exposures (e.g. chemotherapy, rapeseed oil, appetite suppressants).

Primary pulmonary hypertension screening, based on family history and physical exam, includes chest radiograph, ECG, and Doppler echocardiography. Further diagnostic workup includes right heart catheterization to measure pulmonary artery pressures. Treatment of primary pulmonary hypertension includes prostanoids, endothelin antagonists, and phosphodiesterase-5 inhibitors. Treatment success or failure is determined by the patient’s exercise capacity.

The second most likely explanation of this woman’s signs and symptoms is idiopathic pulmonary fibrosis (Choice C) however she is quite young (most cases occur in the sixth decade of life) and one would expect to hear bilateral rales at the bases of the lungs and see clubbing.

Page 50: Internal Medicine- Cardiology

If this were coronary artery disease (Choice B), a much more common cause of dyspnea, we would expect that chest pain would feature more prominently in the symptoms and that the question would mention other cardiovascular disease risk factors.

Systemic lupus erythematosus (Choice E) can lead to pulmonary hypertension, however it is a later consequence of the disease. This woman would have other symptoms and signs of lupus (none of which were listed in the question) by the time she had developed pulmonary hypertension.

A patient with hypertrophic cardiomyopathy with outflow tract obstruction may have a harsh crescendo-decrescendo systolic murmur, which may vary

with various maneuvers.

Please answer the following questions as True or False:

1. Administration of an inotropic drug (e.g. Epinephrine) increases the murmur

2. The Valsalva maneuver increases the murmur

3. Squatting increases the murmur

4. Administration of a veno-dilator (e.g. nitroglycerine) increases the murmur

5. Administration of an arterial vasodilator (e.g. hydralazine) increases the murmur

1) TFTFT 2) FTFFT 3) TTFTT 4) TTTTT 5) FFFTT

The correct answer is choice C

Physical examination: Most patients demonstrate a double or triple apical precordial impulse and a fourth heart sound. Those with intraventricular pressure gradients may have a rapidly rising arterial pulse. The hallmark of obstructive HCM is a systolic murmur, which is typically harsh, diamond-shaped, and usually begins well after the first heart sound. The murmur is best heard at the lower left sternal border as well as at the apex.

Page 51: Internal Medicine- Cardiology

Hemodynamics: In contrast to the obstruction produced by a fixed narrowed orifice, such as valvular aortic stenosis, the pressure gradient in HCM is dynamic and may change between examinations and even from beat to beat. Obstruction occurs due to narrowing of the LV outflow tract by systolic anterior movement (SAM) of the mitral valve against the hypertrophied septum.

The basic mechanisms involved in the production and intensification of the dynamic intraventricular obstruction are:

1. Increased LV contractility

2. Decreased ventricular preload

3. Decreased aortic impedance and pressure (afterload).

Interventions that increase myocardial contractility, such as exercise and sympathomimetic amines (statement 1), and those that reduce ventricular preload, such as the strain phase of the Valsalva maneuver (statement 2), sudden standing, or arterial vasodilators such as hydralazine (statement 5) or venodilators such as nitroglycerin (statement 4), reduce LV end-diastolic volume and, thereby, may cause an increase in the gradient and the murmur. Conversely, elevation of arterial pressure by squatting (statement 3), sustained handgrip, augmentation of venous return by passive leg raising, and expansion of the blood volume (as during pregnancy) all increase ventricular volume and ameliorate the gradient and murmur.

A 14 year old boy presents to your practice to establish care. When rooming the patient, your assistant measures his right brachial artery blood pressure

to be 150/100, which you confirm on repeat testing. On physical exam, you note a III/VI systolic murmur across the left precordium, and his femoral

pulses are delayed and diminished in amplitude when compared to his right brachial artery pulses. You then measure his blood pressure in his right thigh

Page 52: Internal Medicine- Cardiology

at 100/50.

What is the most likely diagnosis?

1) patent ductus arteriosus 2) ventricular septal defect 3) coarctation of the aorta 4) atrial septal defect 5) tetralogy of Fallot

The correct answer is Choice C. This case illustrates the importance of a thorough physical exam when evaluating hypertensive patients, particularly if they are young or have other risk factors for secondary hypertension. Aortic coarctation is present in approximately one in every 3000-4000 live births, and up to 75% of cases also have a bicuspid aortic valve. Although severe cases are usually diagnosed during infancy, patients with less severe cases can remain asymptomatic and undiagnosed into late adulthood. Comparing blood pressure in the right brachial artery and a femoral artery is a good means to identify hemodynamically significant aortic lesions occuring distal to the origin of the brachiocephalic artery. Comparing the timing and ampliture of pulses in the right brachial artery and a femoral artery is similarly helpful. In addition, measurement of pulse oximetry in the right upper extremity and a lower extremity can provide evidence of a right to left shunt in the systemic circulation as may be seen in cases of patent ductus arteriosis in the newborn. Because a non-trivial minority of coarctations occur between the origins of the brachiocephalic and left subclavian arteries (rather than distal to the left subclavian artery), it is important to use the right upper extremity's vasculature for comparison measurements with lower extremity vasculature. The upper extremity hypertension seen in aortic coarctation is caused by decreased blood flow to the kidneys with resultant activation of the renin-angiotensin-aldosterone axis. Patent ductus arteriosis, ventricular septal defects, atrial septal defects, and tetrology of Fallot (Choices A, B, D, and E) are not associated with discrepant upper and lower extremity blood pressures.

A 65-year-old man with a history of type 2 diabetes presents to the emergency department with 4 hours of chest pain. Physical examination reveals a

heart rate of 120 beats per minute and a systolic blood pressure of 62mm Hg. Electrocardiogram reveals ST elevation in leads V1 to V4. The patient

undergoes emergent cardiac catheterization followed by PCI. A pulmonary artery catheter is inserted for hemodynamic monitoring.

Page 53: Internal Medicine- Cardiology

Which of the following hemodynamic patterns is characteristic of true cardiogenic shock?

1) Pulmonary capillary wedge pressure (PCWP) < 15 mm Hg; cardiac index > 2.2 L/min/m2 2) PCWP > 15 mm Hg; cardiac index > 2.2 L/min/m2 3) PCWP < 15 mm Hg; cardiac index < 2.2 L/min/m2 4) PCWP > 15 mm Hg; cardiac index < 2.2 L/min/m2 5) PCWP > maximal pulmonary artery pressure; cardiac index < 2.2 L/min/m2

The correct answer is choice D. Cardiogenic shock is characterized by hypotension and global hypoperfusion in the setting of decreased cardiac output despite normal or increased preload. The cardiac index (CI), which corrects for the normal variation in cardiac output associated with body size, typically ranges from 2.5-4.5 L/min/m2. PCWP approximates left atrial pressure and normally varies from 2-12 mm Hg. PCWPs of less than 5 mm Hg suggest hypovolemia and decreased preload while values over 15 mm Hg reflect increased preload. The hemodynamic criteria defining cardiogenic shock include sustained hypotension, CI less than 2.2 L/min/m2, and PCWP greater than 15 mm Hg. Normal or elevated CIs (Choices A and B) are inconsistent with cardiogenic shock. Normal or decreased PCWPs (Choice C) are also inconsistent with cardiogenic shock. A measured PCWP exceeding maximal pulmonary artery pressure (Choice E) suggests a reversed pressure gradient and retrograde flow through the pulmonary artery. This is a physiologic impossibility and likely represents ―overwedging,‖ in which the pulmonary artery catheter tip is pressed against the vascular wall when the balloon is inflated. In this instance, the measured PCWP does not accurately approximate left atrial pressure.

A 19-year-old college student presents with chest pain and shortness of breath. One week previously, she had a self-limited upper respiratory infection.

The chest pain is central in location and varies with respiratory phase. The patient is tachypneic, and S1 and S2 sound muffled on exam despite the

patient’s thin body habitus. An EKG demonstrates diffuse ST elevations.

All of the following findings are commonly associated with hemodynamically significant pericardial tamponade EXCEPT :

Page 54: Internal Medicine- Cardiology

1) Elevated jugular venous pressure (JVP) 2) Hypertension 3) Kussmaul’s sign 4) Low voltage QRS complexes 5) Pulsus paradoxus

The correct answer is Choice B. The patient presents with signs and symptoms suggestive of infectious pericarditis with possible pericardial effusion. In such patients, it is imperative to have a high index of suspicion for the possibility of tamponade, which may be life threatening in the absence of intervention. All of the signs listed as possible answers are associated with tamponade with the exception of pulmonary edema. Elevated JVP (Choice A) is seen in 76% of patients with tamponade. Hypotension (not hypertension as is Choice B) is seen in 26% of patients with tamponade. Kussmaul’s sign (Choice C) is the observation of increased JVP during inspiration. It is also seen in approximately 26% of patients with tamponade, generally only in the setting of constrictive disease. Pulsus paradoxus (Choice E) represents an exaggeration of the normal variation in SBP associated with respiratory phase. It may be detected by slowly deflating the blood pressure cuff while listening for the first Korotkoff sound. It will initially become audible only while the patient is exhaling and later become audible regardless of respiratory phase. The clinician then calculates the difference between the SBP at which the first Korotkoff sound is heard intermittently and the SBP at which it is heard throughout the respiratory cycle. Some authorities use a difference of 10mm Hg to define pulsus paradoxus while others use 12mm Hg. Using the 10mm Hg criterion, pulsus paradoxus is present in approximately 82% of patients with tamponade.

Other signs of tamponade include ST segment changes on the electrocardiogram, which may also show low voltage QRS complexes (choice D), as well as general signs & symptoms of shock (such as tachycardia, breathlessness and decreasing level of consciousness).

Causes of increased pericardial effusion include hypothyroidism, physical trauma (either penetrating trauma involving the pericardium or blunt chest trauma), pericarditis (inflammation of the pericardium), iatrogenic trauma (during an invasive procedure), and myocardial rupture.

A 30-year old woman is complaining of chest pain and palpitations. On auscultation, first heart sound is normal, a midsystolic and a late systolic murmur

Page 55: Internal Medicine- Cardiology

is present. On the EKG, T-wave inversion is present in leads II, III and aVF. Which of the following statement is true about her condition?

1) Echocardiography shows superior displacement of both mitral leaflet by 2.5 mm above the plane of annulus as well as excessive chordal length and motion

2) The cause of this woman’s chest pain is partial occlusion of coronary ostia 3) An exercise ECG testing is likely to be positive 4) This woman is not at an increased risk of infective endocarditis and prophylactic antibiotics are not recommended 5) The click and murmur are expected to occur earlier when the patient is in supine position with legs elevated

The correct answer is choice A The most specific criterion for diagnosis of MVP syndrome is superior displacement of both mitral leaflet by more than 2 mm above the plane of annulus in the long axis. Other echocardiographic criteria are diffuse leaflet thickening and redundancy, excessive chordal length and motion, and evidence of ruptured chords, in addition to prolapse of leaflet segments (choice A). The majority of MVP patients remain asymptomatic throughout life. Patients may complain of syncope, presyncope, palpitations, chest discomfort. Symptoms of low cardiac reserve may occur if MR is severe. Chest discomfort may be typical of angina pectoris but is more often atypical, prolonged and not clearly related to exertion. In MVP syndrome, prolapsed of mitral valve places excessive abnormal tension on the papillary muscles and lead to ischemia and chest pain (choice B). Although MVP syndrome is often associated with T-wave changes in inferior leads, ischemic response to exercise is only occasionally observed (choice C). Any manoeuvre that increases LV volume such as an increase in the impedance to LV emptying, an increase in venous return, a reduction of myocardial contractility, or bradycardia, both the click and the onset of the murmur will be delayed (choice E). MVP is the most common cardiac condition predisposing patients to infective endocarditis and thus appropriate treatment should be given (choice D).

Page 56: Internal Medicine- Cardiology

A 35-year-old man presents to cardiology out-patioent department for evaluation of his dyspnea. He was asymptomatic 12-months back when he experienced dyspnea on exertion. It has increased progressively and at present he cannot walk more than 15 ft at stretch. On examination, cyanosis and clubbing was noticed in his toes but not the fingers. He is hypoxic with oxygen saturation of 85% on room air. Cardiac examination reveals a harsh machinery murmur that is continuous throughout systole and diastole with a palpable thrill. There is late systolic accentuation of the murmur at the upper left sternal angle. What is the most likely cause of this patient’s murmur?

1) Anomalous pulmonary venous return 2) Coarctation of the aorta 3) Patent ductus arteriosus 4) Tetralogy of Fallot 5) Ventricular septal defect

The correct answer is choice C The ductus arteriosus is a blood vessel present in embryonic life which connects the pulmonary artery to the aorta just distal to the left subclavian artery, shunting blood from the fluid-filled lungs of the fetus. After birth, the ductus arteriosus closes as blood starts circulating through the low-resistance pulmonary vascular bed. If the ductus arteriosus fails to close after birth, a left-to-right shunt develops between the aorta and the pulmonary vasculature. Because the pressure in the aorta is greater than that of the pulmonary artery throughout the cardiac cycle, the murmur of a patent ductus arteriosus is a continuous murmur. There is late systolic accentuation of the murmur at the upper left sternal angle. The murmur is described as ―machinery‖-like, and often a palpable thrill is present. If Eisenmenger syndrome occurs, as in this patient, the shunt changes directional flow and becomes a right-to-left shunt as a result of pulmonary hypertension. That is when patients will become cyanotic. Because of the anatomic location of the ductus arteriosus below the level of the left subclavian artery, a characteristic of Eisenmenger syndrome in those with patent ductus arteriosus is cyanosis and clubbing of the toes but not the fingers. Total anomalous pulmonary venous return occurs when all four pulmonary veins drain into the systemic venous circulation. This condition is fatal soon after birth unless there is an atrial or ventricular septal defect or a patent foramen ovale. Most patients with this condition are identified shortly after birth because of cyanosis (choice A). Coarctation of the aorta is a relatively common congenital abnormality in which there is a stricture of the aorta near the insertion site of the ligamentum arteriosus (the remnant of the ductus arteriosus). A patient with coarctation of the aorta frequently presents with headache. Upper extremity hypertension

Page 57: Internal Medicine- Cardiology

is present in association with low blood pressures in the lower extremities. Patients may also complain of claudication in the lower extremities (choice B). Tetralogy of Fallot is a congenital heart disease syndrome with ventricular septal defect, right-ventricular outflow obstruction, aortic override of the ventricular septal defect, and right-ventricular hypertrophy. This defect is almost always identified and corrected during childhood (choice D).

Ventricular septal defect results in left to- right shunt and a holosystolic murmur rather than a continuous murmur (choice E).

A 70-year old asymptomatic male patient presents to the outpatient clinic for screening purposes. On physical examination, his blood pressure was 140/90 mm Hg in both arms measured on two different occasions with no other remarkable findings. Lab investigations did not show any abnormal values. Which of the following is the class of antihypertensive agent most appropriate for initiating treatment of hypertension in this patient?

1) Angiotensin receptor blocker 2) Calcium channel blockers 3) Alpha blockers 4) Thiazide diuretics 5) Beta blockers

The correct answer is choice D

According to JNC-7 guidelines, thiazide diuretics are recommended for initial therapy of most patients of hypertension either alone or in combination with another class. The algorithm below shows the treatment approach for the treatment of stage 1 hypertension. These guidelines recommend thiazide diuretics or angiotensin-converting enzyme (ACE) inhibitors as first-line agents for blood pressure control in most patients.

Page 58: Internal Medicine- Cardiology
Page 59: Internal Medicine- Cardiology

A patient presents to emergency room with chest pain started 5 hours ago. He is diagnosed with acute coronary syndrome. Before admission he got 3

tables of nitroglycerin sublingually and aspirin orally. Currently his blood pressure is 140/80 mm Hg, heart rate is 86/min, regular. On supplemental

oxygen the respiratory rate is 16/min and arterial oxygen saturation (SaO2) is 94 percent. Lungs are clear on auscultation. After administration of

morphine sulfate intravenously his chest pain is completely relieved.

Which beta-adrenergic blocker is best choice for the patent initially?

1) Metoprolol tartrate 5 mg intravenously over 1 to 2 minutes 2) Acebutolol 200 mg orally 3) Atenolol 25 mg orally 4) Beta blocker should be initiated for this patient only after 24 hours of onset of pain 5) Beta blocker cannot be initiated in the patient due to presence of contraindications

The correct answer is Choice C Multiple clinical trials on a broad range of patients with both ST elevation and non-ST elevation myocardial infarction (MI), including those with and without revascularization, showed that beta blockers reduce short-term complications and improve long-term survival.

In the absence of contraindications, the guidelines published by the ACC/AHA and the European Society of Cardiology recommend initiation of oral beta blockers within the first 24 hours in all patients with acute coronary syndrome. (Choice D).

Major contraindications to beta blocker therapy include active bronchospasm, severe bradycardia, heart block greater than first degree (unless the patient has a permanent pacemaker), pulmonary edema, and hypotension with or without shock. None of these contraindications present is our patient (Choice E).

Patients who do not receive a beta blocker during the first 24 hours because of early contraindications should be reevaluated for beta blocker candidacy for subsequent therapy. In the absence of contraindications, the patients who have sustained an acute MI should continuing oral beta blockers indefinitely.

Page 60: Internal Medicine- Cardiology

The beta blockers without intrinsic sympathomimetic activity (ISA) are preferred due to significantly more benefits for patients compared to agents without ISA (acebutolol, oxprenolol and pindolol) (Choice B).

Patients with hypertension on admission or those needing immediate beta blockade for other reasons, such as atrial fibrillation with rapid ventricular response, may receive intravenous beta blockade with metoprolol tartrate (5 mg increments by slow intravenous administration repeated every five minutes for a total initial dose of 15 mg) (Choice A) Patients who tolerate this regimen should then receive oral therapy of metoprolol tartrate 25 to 50 mg every six hours.

For our patient the best initial regime would be oral metoprolol or atenolol 25-50 mg, titrating upward as needed. The full doses are metoprolol tartrate (immediate release preparation) 50 to 100 mg twice daily or metoprolol succinate (extended release preparation) 100 mg daily or atenolol 50 to 100 mg daily or divided twice daily. Bradycardia and hypotension are the most common limitations to achieving the full dose. The goals are to reduce the heart rate less than 70 beats/min while maintaining a systolic pressure above 90 mmHg.

During what period after onset of ST-elevation myocardial infarction do the following described ECG changes develop:

The ST segment gradually returns to the isoelectric baseline, the R wave amplitude becomes markedly reduced, the Q wave deepens and the T wave

becomes inverted?

1) During first 24 hours 2) During first two weeks 3) During first two months 4) ST segment never returns to the isoelectiric baseline 5) Before normalization of the creatine kinase level

The correct answer is Choice B Myocardial infarction is defined as a pathologic event caused by myocardial ischemia in which there is evidence of myocardial injury or necrosis. The electrocardiogram (ECG) is an important and sometimes central tool used to establish the diagnosis of myocardial ischemia or infarction. New abnormalities in the ST segment and T waves represent myocardial ischemia and may be followed by the formation of Q waves. However, remember that the electrocardiogram may be normal or nonspecific in a patient with either ischemia or infarction.

Page 61: Internal Medicine- Cardiology

The published guidelines established specific ECG criteria for the diagnosis of ST elevation myocardial infarction, which include 2 mm of ST segment elevation the precordial leads for men and 1.5 mm for women (who tend to have less ST elevation) and greater than 1mm in other leads. The findings on the ECG depend upon several characteristics of the ischemia or infarction such as duration, size and localization.

The earliest change, which is not frequently seen, is symmetric, hyperacute T waves (defined as amplitude more than 50 percent of the R wave in the same lead) in at least two contiguous leads, which reflects a localized increase in plasma potassium concentration.

During first 24 hours (Choice A) there is initially elevation of the J point and the ST segment retains its concave configuration. Over time the ST segment elevation becomes more pronounced and the ST segment changes its morphology, becoming more convex or rounded upward. The ST segment may eventually become indistinguishable from the T wave; the QRS-T complex can actually resemble a monophasic action potential. An initial Q wave develops and there is a loss of R wave amplitude as the ST segment becomes elevated.

During next two weeks (Choice B) the ST segment gradually returns to the isoelectric baseline, the R wave amplitude becomes markedly reduced, the Q wave deepens and the T wave becomes inverted.

Two months later (Choice C) after ST-eleveation myocardial infarction abnormal Q waves may be seen on ECG sometimes along with inverted T waves. An abnormal Q wave is any Q wave in leads V1 to V3 or a Q wave ≥30 msec in leads I, II, aVL, aVF, or V4 to V6 and they must be present in any two contiguous leads and ≥1 mm in depth. The location of these changes is dependent upon the location of the postmyocardial scar.

If there is persistent elevation of the ST segment on ECG a left ventricular aneurism should be suspected in a patient that usually develops as a complication of a large anterior myocardial infarction (Choice D).

Creatine kinase level rises during first 3-6 hours of myocardial infarction (the same as troponins), it peaks between 18-24 hours of onset and then decreases to normal during next 48 hours. The described ECG changes will not usually occur during such a short period of time (Choice E).

Page 62: Internal Medicine- Cardiology

An asymptomatic 24 year old primigravid woman at 36 weeks gestation comes for a routine prenatal visit. A grade 2/6, ejection systolic murmur is heard

at the 2nd left intercostal space. The S2 varies with inspiration, and the pulmonic component is soft; no sound was heard in diastole.

Which of the following is the most likely diagnosis?

1) Atrial septal defect 2) Flow murmur 3) Ruptured sinus of valsalva 4) Patent ductus arteriosus 5) Pulmonary valve stenosis

The correct answer is choice B

The auscultation in a patient of atrial septal defect (choice A) reveals normal or split first heart sound, with accentuation of the tricuspid component. Increased flow across the pulmonic valve results in midsystolic pulmonary outflow murmur. The second heart sound is widely split and is relatively fixed in relation to respiration. A mid-diastolic rumbling murmur may be present which reflects increased flow across the tricuspid valve. Heart murmurs are common in pregnancy and usually reflect increased flow across structurally normal valves secondary to hemodynamic changes of pregnancy. This increased flow results in flow murmur (choice B). Rupture sinus of valsalva (choice C) may have similar auscultatory findings but the disease is itself rare and is extremely rare in an asymptomatic pregnant patient. Even unruptured aneurysm can obstruct the cardiac outflow or cause heart block. Ruptured aneurysm may result in acute onset of congestive heart failure, arrhythmia, cardiac tamponade and even sudden death. Ruptured sinus of valsalva in asymptomatic pregnant patient is extremely rare and there are only few reported cases. In most adults with patent ductus arteriosus (choice D), a gradient and shunt from aorta to pulmonary artery persist throughout the cardiac cycle, which results in a characteristic thrill and continuous "machinery" murmur with late systolic accentuation at the upper left sternal edge. In pulmonary valve stenosis (choice E) harsh systolic crescendo-decrescendo murmur is heard.

Page 63: Internal Medicine- Cardiology

A 65-year-old man has had congestive heart failure with increasing pulmonary congestion and edema for the past year. He had been previously healthy

all his life with no major illnesses. On physical examination his blood pressure is 125/85 mm Hg and he is afebrile. A chest radiograph shows

cardiomegaly with a prominent left heart border and pulmonary edema. Laboratory studies show a serum glucose of 95 mg/dL and total serum

cholesterol of 175 mg/dL. His serum creatine kinase is not elevated.

Which of the following underlying diseases is he most likely to have?

1) Alcoholic cardiomyopathy 2) Calcified bicuspid aortic valve 3) Tricuspid valve endocarditis 4) Aortic dissection 5) Amyloidosis

The correct answer is choice B.

This is a case of congestive heart failure most probably due to a calcified bicuspid aortic valve. The diagnosis is made by exclusion of other causes as shown below.

The normal aortic valve has 3 equal-sized cusps with 3 lines of coaptation. The bicuspid valve is composed of 2 leaflets or cusps, usually of unequal size. The larger leaflet is referred to as the conjoined leaflet. It may be associated with coarctation or interrupted aortic arch, patent ductus arteriosus, Turner syndrome (bicuspid aortic valve occurs in 30% of patients) or Williams syndrome (bicuspid aortic valve associated with supravalvular aortic stenosis occurs in 11.6% of cases).

With degeneration of aging valves, sclerosis and calcification can occur leading to aortic stenosis. In this case there is normal total serum cholesterol (175 mg/dL), so the stenosis is due to calcification. This stenosis would lead to progressive congestive heart failure (increasing pulmonary congestion and edema, cardiomegaly with a prominent left heart border on the chest radiogarph).

This image shows bicuspid aortic valve with unequal cusp size. Note eccentric commissure and raphe.

Page 64: Internal Medicine- Cardiology

Alcoholic cardiomyopathy (choice A) can lead to congestive heart failure yet there must be a clear history about alcohol intake to make such diagnosis. Also alcoholic cardiomyopathy is usually associated elevated total serum cholesterol, hypertension and may be hypoglycemia (all are not seen in this case).

Tricuspid valve endocarditis (choice C) is usually presented by fever, chills and other classic signs of endocarditis as petechiae, splinter hemorrhages and Osler nodes. Tricuspid affection would not be presented by pulmonary congestion unless it is preceded by manifestations of systemic congestion as jugular venous distention, peripheral edema, enlarged tender liver and jaundice.

Aortic dissection (choice D) is usually presented by ripping or tearing pain in the intrascapular area with abrupt onset and not assuming the log-term course seen in this case.

Cardiac amyloidosis (choice E) typically causes diastolic dysfunction leading to congestive heart failure, however, it is almost always characterized by

Page 65: Internal Medicine- Cardiology

hypotension and orthostatic hypotension. It would lead to diffuse cardiac enlargement not the left border only. Primary manifestations of amyloidosis should be evident as peripheral edema, hepatomegaly, purpura, peripheral neuropathy, carpal tunnel syndrome, and macroglossia.

A 67-year-old woman presents to her primary care physician with complaints of progressive dyspnea, fatigue, generalized malaise, and lower extremity

edema. Three weeks previously, she had undergone mitral valve replacement with a St. Jude’s mechanical prosthesis for mitral stenosis/mitral

regurgitation. She has been closely adherent to her warfarin regimen with therapeutic INRs.

Physical examination reveals an afebrile patient with tachycardia (heart rate, 120 bpm), systolic blood pressure of 90 mm Hg with a pulsus paradox of 20

mm Hg, respiratory rate of 26 breaths/min, and jugular venous distension to the angle of the jaw. Cardiac examination reveals muted heart sounds, a

friction rub, no mitral regurgitation murmur, and crisp prosthetic valve sounds. Extremities are cool with 2+ edema. The ECG reveals sinus tachycardia

and diffuse ST segment elevations. She is urgently admitted to the hospital.

Which of the following tests would provide the most useful information to arrive at a prompt diagnosis?

1) Chest radiograph 2) Computed tomography of the chest 3) Magnetic resonance imaging of the chest 4) 2-D echocardiography with Doppler 5) Ventilation/perfusion scan

The correct answer is Choice D. The patient's presentation is strongly suggestive of pericarditis with pericardial effusion and tamponade. Late pericardial effusion with tamponade is a well recognized complication of valvle replacement. In a recent study of 421 patients who underwent valve replacement, 17% had pericardial effusions on post operative day 20, and 11% of those with effusions went on to develop tamponade. Anticoagulant use is associated with increased risk of postoperative effusion. Echocardiography is the imaging study of choice to confirm the diagnosis, and if it's indicated, pericardiocentesis can be performed at the same time as the echocardiogram. Chest x-ray, computed tomography, and magnetic resonance imaging (Choices A, B, and C) all have the potential to demonstrate the presence of

Page 66: Internal Medicine- Cardiology

pericardial effusion. However, echocardiogaphy is preferrable. It provides more information about the hemodynamic consequences of pericardial effusion and tamponade then the other imaging modalities mentioned. It can also be used to guide drainage and does not expose the patient to ionizing radiation. It is always prudent to consider the possibility of pulmonary embolism (PE) in post-operative patients presenting with dyspnea and new symptoms consistent with heart failure. A ventilation/perfusion scan (Choice E) is an acceptable diagnostic study to refine the probability of pulmonary embolism. However, the patient's history and physical exam are much more suggestive of pericardial effusion and tamponade than PE, and new onset venous thromboembolic disease is unlikely in a patient taking warfarin at therapeutic doses.

All of the following statements are true regarding AV bock includes except: 1) In first degree AV block the intensity of first heart sound is increased 2) The conduction abnormality in Mobitz type I second degree heart block with normal QRS duration is almost always at the level of AV node,

proximal to bundle of His 3) In typical Mobitz type I second degree heart block, the PR interval progressively increases over consecutive beats, until a beat is dropped 4) In Mobitz type II second degree heart block, the PR intervals are constant prior to the non conducted P wave 5) The ventricular escape rate in acquired complete heart block is usually less than 40 beats per minute

The correct answer is choice A An AV block exists when atrial impulses are conducted to the ventricles with abnormal delay or are not conducted at all. It has been classified by severity into three categories. In first-degree AV block, all atrial impulses are conducted but the conduction time (seen as prolonged PR interval >0.20 second in ECG) is increased. In first-degree heart block, the delay between atrial and ventricular contraction allows the leaflets of the mitral and tricuspid valves to drift toward a partially closed position prior to ventricular systole. Therefore, the intensity of the first heart sound is diminished. In second-degree heart block, some impulses fail to conduct from the atria to the ventricles. Second-degree block is divided into two groups: Mobitz type I and Mobitz type II. In the former type (also termed Wenckebach block), the PR interval progressively increases (and the R-R interval usually progressively shortens) until an atrial impulse fails to conduct to the ventricles (choice C). In Mobitz type II block, the PR intervals are constant, and without warning there is intermittent failure of an atrial impulse to conduct to the ventricles (choice D).

Page 67: Internal Medicine- Cardiology

Third-degree heart block is present when all atrial impulses fail to conduct to the ventricles such that the atrial and the ventricular rhythms are independent of one another. Mobitz type I second-degree heart block with a normal QRS duration almost always occurs in the AV node, proximal to the His bundle(choice B). Conversely, type II second-degree heart block, especially when accompanied by a bundle branch block QRS morphology, usually reflects a more serious abnormality in the His-Purkinje system. In third-degree heart block, there is complete AV dissociation. As a result, the ventricular rate is independent of atrial rate, and governed by an independent ventricular escape pacemaker. In acquired forms of third-degree heart block, the ventricular rate is usually less than 40beats/min (choice E). The ventricular rate tends to be faster in patients with congenital complete heart block, about 50beats/min.

A 3 year old male is brought to the physician by his mother after she noted multiple bruises on his body after picking him up from daycare, though the

caregivers at that facility denied any recent trauma. Two weeks ago, the patient had three days of cough and low-grade fever that resolved without

antibiotic therapy, but since that time he has been well. Past medical history is otherwise unremarkable. The patient is up-to-date on all required

immunizations. On physical exam, the patient is afebrile and playing happily at his mother’s feet in the examination room. There are no peritonsillar

exudates. Tympanic membranes are clear. There is no abdominal tenderness or organomegaly. Examination of the skin shows diffuse and numerous

petechiae and purpura. Laboratory analysis reveals the following:

WBC 7,500/mcL

Hemoglobin 14 g/dL

Hematocrit 42%

Platelets 27,000/mcL

Total bilirubin 0.7 mg/dL

AST 22 U/L

ALT 18 U/L

alkaline phosphatase 120 U/L

LDH 186 U/L

PT 11 s

PTT 22 s

Which of the following is the most appropriate next step in the management of this patient?

1) Bone marrow biopsy

Page 68: Internal Medicine- Cardiology

2) Immediate platelet transfusion 3) Intramuscular ceftriaxone 4) Oral prednisolone 5) Notify police and child protective services

The correct answer is choice D.

This patient has idiopathic thrombocytopenic purpura (ITP). ITP is a diagnosis of exclusion made by finding low platelets and ruling out other diseases and toxic exposures that result in decreased platelet counts.

This is an autoimmune disease caused by antibody-mediated destruction of platelets, so first line therapies are high-dose corticosteroids like prednisolone (choice A) as well as intravenous immunoglobulin (IVIg). Patients refractory to these treatments may require more high-powered immunosuppressant drugs or splenectomy.

A few teaching points about ITP and thrombocytopenia:

1) There are two groups of patients who get ITP: children from 2-4, and adults (usually women) in their 20s-40s. Children in general do much better with the disease: over 80% of children will have a spontaneous remission of their disease, while only 20% of adults have a sustained remission.

2) It is extremely important to rule out other diseases before you make the diagnosis of ITP. Here, the patient’s CBC is normal with the exception of low platelets – thus, you don’t have to worry about leukemia, which would be another common cause of this presentation in a young child. DIC and TTP should also be excluded by looking at a peripheral blood smear for schistocytes or laboratory evidence of microangiopathic hemolytic anemia (increased LDH or bilirubin or decreased haptoglobin and hematocrit). Patients with ITP generally look like this patient – otherwise well except for their thrombocytopenia.

3) Up to 85% of pediatric patients will have a history of an antecedent infection. This is a common clue to the diagnosis on standardized medical exams.

Page 69: Internal Medicine- Cardiology

A bone marrow biopsy (choice A) is not required. Had this child presented with thrombocytopenia and anemia or leukocytosis/leukopenia, leukemia would have to be excluded with a bone marrow biopsy. However, with only one cell line involved, ITP is the overwhelmingly likely diagnosis.

Immediate platelet transfusion (choice B) is not indicated at this time. The risk of spontaneous bleeding does not increase until the platelet count falls below 20,000, and patients with ITP in particular often tolerate even lower platelet counts without spontaneous bleeding. In the absence of serious active bleeding, you should hold off on giving any transfusions – remember, treat the patient, not the numbers!

Ceftriaxone (choice C) is a third-generation cephalosporin that is used to treat serious bacterial infections like meningitis in childhood. Thrombocytopenia can occur in patients with sepsis, but this child is well-appearing except for his bruising.

Notifying the police and child protective services (choice E) is appropriate whenever there are significant concerns for child abuse. However, this patient's bruising is clearly caused by his thrombocytopenia.

A 78 year old male, who suffered an antero-lateral myocardial infarction (MI) in 2007, is a patient of yours at the weekly heart failure clinic in the university hospital. His main complaint is that of ongoing swelling of his bilateral legs and feet for the past 2 month. He is presently taking Ramipril 5 mg BID, Hydrochlorthiazide 12.5 mg OD and Asprin 160 mg OD. His physical status is relativiely good and he claims that he is able to climb two flights of stairs without any dyspnea. Clinical examination is remarkable for a heart rate of 98 beats/min and regular, blood pressure of 174/98 mmHg. His jugular venour pressure (JVP) was not raised, but he had 2+ pitting pedal edema up to shins. Auscultation of heart reveals an isolated S3 with no associated gallop rhythms or murmurs. Lungs are clear to the bases. Routine investigations including CBCD, creatinine, urea, lipid profile, blood sugar and electrolytes are within normal limits. ECG is suggestive of prior antero-lateral infarct with no sign of atrial fibrillation. An ECHO was done which shows global hypokinesia, dialated left ventricle, a small chronic mural thrombus attached to the apex of left ventricle and an ejection fraction (EF) of 25%. All of the following statements are true for this patient except:

1) He should be started on a blocker, preferably Carvedilol 2) Implantation of an ICD is an appropriate measure 3) He should be started on an aldosterone antagonist 4) No anticoagulation is required for this patient 5) Revascularisation will offer no benefit in this individual

Page 70: Internal Medicine- Cardiology

The correct answer is choice C

This question is designed to test the student’s knowledge of the treatmenst for heart failure. This patient is suffering from congestive cardiac failure due to a past MI. He is asymptomatic and hence his functional status would fall under NYHA Class I (see below for NYHA classifications).

Aldosterone antagonists have been shown to provide mortality benefits in patients of heart failure with NYHA class III/IV symptoms. Our patient is asymptomatic and hence addition of Spirinolactone will give no added benefit (choice C) .

The MERRIT trial firmly established the role of β-blockers in established heart failure. Addition of a β-blocker to the current regimen of medicines will enhance survival in this patient and hence addition of a β-blocker would be an appropriate next move (choice A). The COMET study did a comparison head-on between Carvedilol and Metaprolol. The annual mortality rate was reduced from 10% in the metoprolol group to 8.3% in the carvedilol group, which translated into a median 1.4 years prolongation of survival in this trial.

Implantation of prophylactic ICD in a sub group of patients with ejection fraction less than 30% in presence of prior MI or EF<35% with symptoms of heart failure is considered a Class IIa recommendation by the American college of cardiology, hence implantation of a ICD is also an appropriate move (choice B).

The ECHO also shows a small mural thrombus in the left ventricular cavity. This is accompanied by dilated left ventricle and CHF with low ejection fraction. A number of different trials have been conducted to study efficacy of oral anticoagulation on thrombo-embolic phenomenon in the setting of dilated chamber and reduce EF. However none have conclusively proved the superiority of oral anticoagulation in the above mentioned setting especially in the presence of a chronic mural thrombus. However if this same thrombus were to develop post MI then he would be an ideal candidate for oral anticoagulation with target INR of 2-3 (choice D).

This patient is asymptomatic with no complaints of angina or baseline changes suggestive of ischaemia in the ECG. Hence he does not require any investigation to assess patency of coronaries. Accordingly there is no requirement for revascularisation procedure in this patient (choice E).

Page 71: Internal Medicine- Cardiology

Table 1: The Stages of Heart Failure – NYHA Classification

The development of conduction disturbances due to myocardial infarction and arrhythmia depends on the vascular supply of the different components of

the heart conduction system.

All statements are correct about the blood supply of the conduction system of the heart EXCEPT:

1) Sinoatrial node is is supplied by the right coronary artery in 60 percent of patients and by the left circumflex artery in 40 percent of patients 2) Atrioventicular node supplied mostly by the right coronary artery 3) Main or proximal left bundle branch is mostly supplied by the left anterior descending artery 4) Left posterior and anterior fascicles are supplied by the left anterior descending artery 5) Right bundle branch is supplied by the left anterior descending artery

Page 72: Internal Medicine- Cardiology

The correct answer is Choice E

The vascular supply of the different components of the conduction system includes:

(Choice A) Sinoatrial node is supplied by the right coronary artery in 60 percent of patients; by the left circumflex artery in 40 percent.

(Choice B) AV node is supplied by the right coronary artery in 90 percent (AV nodal branch); by the left circumflex artery in 10 percent of patients.

His bundle is supplied by the right coronary artery (AV nodal branch) with a minor contribution from the septal perforators of the left anterior descending artery.

(Choice C) Main or proximal left bundle branch is mostly supplied by the left anterior descending artery, particularly for the initial portion. There may be some collateral flow from the right coronary artery and left circumflex artery systems.

(Choice D) Left posterior fascicle — The proximal part of the left posterior fascicle is supplied by the AV nodal artery and, at times, by septal branches from the left anterior descending artery. The distal portion has a dual blood supply from both anterior and posterior septal perforating arteries.

(Choice D) Left anterior fascicle — The left anterior and mid-septal fascicles are supplied by septal perforators of the left anterior descending artery and, in about one-half of subjects, by the AV nodal artery

(Choice E) Right bundle branch — The right bundle branch receives most of its blood supply from septal perforators from the left anterior

Page 73: Internal Medicine- Cardiology

descending artery coronary artery, particularly in its initial course. It also receives some collateral supply from either the right coronary artery and left circumflex artery systems, depending upon the dominance of the coronary system

In inferior myocardial infarction conduction disturbance such as sinus bradycardia, Mobitz type I (Wenckebach), and complete heart block are commonly seen, since the SA node, AV node, and His bundle are primarily supplied by the right coronary artery. The incidence of complete heart block is significantly higher among patients with an inferior MI than among those with an anterior MI.

Complete heart block with inferior myocardial infarction generally results from an intranodal lesion. It is associated with a narrow QRS complex, and develops in a progressive fashion from first to second to third degree block. It often results in an asymptomatic bradycardia (40 to 60 beats/min) and is usually transient, resolving within five to seven days.

In anteroseptal myocardial infarction conduction disturbances are more serious and the degree of arrhythmic complications is usually directly related to the extent of infarction. For example, complete heart block with anterior myocardial infarction generally occurs abruptly in the first 24 hours. It can develop without warning or may be preceded by the development of right bundle branch block with either left anterior fascicular block or left posterior fascicular block (bifascicular or trifascicular block). The escape rhythm is wide and unstable and the event is associated with a high mortality (approximately 80 percent) from both arrhythmias and pump failure. Heart block in this setting is thought to result from extensive necrosis that involves the bundle branches traveling within the septum .

A 72-year old male presents to cardiac out-patient department for a routine examination. He is asymptomatic at present. He has a past medical history of hypertension and hypercholesterolemia. He is also a chronic smoker. He is taking hydrochlorthiazide 25 mg daily, atenolol 25 mg daily, and simvastatin 40 mg nightly. On examination, pulse rate is 70 per minute and blood pressure is 130/80. On auscultation, bilaterally vesicular breath sounds are heard. Cardiac auscultation is unremarkable. During abdominal examination, a pulsatile mass is felt just left to umbilicus. CT scan of abdomen confirms a diagnosis of abdominal aortic aneurysm. The aneurysm is located infrarenally and measures 4.8 cm. All of the following are true about the patient’s condition except:

1) The 5-year risk of rupture of an aneurysm of this size is 1-2% 2) Abdominal ultrasound is useful for screening the high risk patients 3) An abdominal ultrasound can delineate the size of the aneurysm but is unable to detect the mural thrombi 4) Surgical or endovascular intervention is warranted for rapidly expanding aneurysm or if the patient becomes symptomatic 5) Surgical or endovascular intervention is indicated for the aneurysms >5.5 cm even if asymptomatic

The correct answer is choice C

Page 74: Internal Medicine- Cardiology

Abdominal aortic aneurysms (AAAs) ≥4 cm affects 1 to 2% of men older than age 50. At least 90% of all abdominal aortic aneurysms >4.0 cm are related to atherosclerotic disease, and most of these aneurysms are located below the level of the renal arteries. Most AAAs are asymptomatic and are found incidentally on physical examination. Most AAAs are located infrarenally, and recent data suggest that an uncomplicated infrarenal AAA may be treated with endovascular stenting instead of the surgical repair. The predisposing factors for AAA are the same as those for other cardiovascular disease, with over 90% being associated with atherosclerotic disease. Prognoses of these aneurysms depend on both the size of the aneurysm and the severity of coexisting coronary artery and cerebrovascular disease. The risk of rupture increases with the size of the aneurysm: the 5-year risk for aneurysms <5 cm is 1–2% (choice A), whereas it is 20–40% for aneurysms >5 cm in diameter. The formation of mural thrombi within aneurysms may predispose to peripheral embolization. Serial ultrasonography or CT imaging is imperative, and all aneurysms larger than 5.5 cm warrant intervention because of the high mortality associated with repair of ruptured aortic aneurysms. Abdominal ultrasound is useful for serial documentation of aneurysm size and can be used to screen patients at risk for developing aortic aneurysm (choice B). An abdominal ultrasound can also delineate the transverse and longitudinal dimensions of an abdominal aortic aneurysm and may detect mural thrombus (choice C). Operative repair of the aneurysm and insertion of a prosthetic graft are indicated for abdominal aortic aneurysms of any size that are expanding rapidly or are associated with symptoms (choice D). For asymptomatic aneurysms, operation is indicated if the diameter is >5.5 cm (choice E).

A 19-year-old woman has been experiencing increasing malaise for the past 5 months. Physical examination reveals a cardiac murmur with a mid

systolic click. An echocardiogram demonstrates mitral insufficiency with upward displacement of one leaflet. The aortic root is dilated to 4 cm. The

patient has a dislocated right ocular crystalline lens.

A year later, the patient dies suddenly and unexpectedly. The medical examiner finds a prolapsed mitral valve with elongation, thinning, and rupture of

Page 75: Internal Medicine- Cardiology

chordae tendineae.

A mutation involving which of the following genes is most likely to be present in this patient?

1) Beta-myosin 2) CFTR 3) Cardiac Myosin-Binding Protein C 4) Fibrillin 5) Spectrin

The correct answer is choice D.

This patient had Marfan syndrome, a fibrillinopathy associated with over 600 mutations in the fibrillin-1 (FBN1) gene. Fibrillin-1 constitutes the backbone of extracellular matrix microfibrils. FBN1 mutations reduce the availability of fibrillin-1 to form microfibrils, probably weakening the elastic fibers and causing overactivation of TGF-beta growth factors, which leads to the signs and symptoms of Marfan syndrome.

Some of the key features of Marfan syndrome include aortic root dilatation/dissection, ectopia lentis (dislocated lens of the eye), and a cardiac murmur with an ejection click related to mitral prolapse and regurgitation.

Marfan syndrome affects males and females equally,and the mutation shows no ethnic or geographical bias. Estimates indicate that approximately 1 in 3,000 to 5,000 individuals have Marfan syndrome. Each parent with the condition has a 50% risk of passing the genetic defect on to any child due to its autosomal dominant nature. Most individuals with Marfan syndrome have another affected family member - approximately 15–30% of all cases are due to de novo genetic mutations- such spontaneous mutations occur in about 1 in 20,000 births.

There is no cure for Marfan syndrome, but life expectancy has increased significantly over the last few decades, and clinical trials are underway for a promising new treatment. The syndrome is treated by addressing each issue as it arises, and, in particular, considering preventive medication, even for young children, to slow progression of aortic dilation.

Mutationsof the beta-myosin gene (Choice A) are involved in hypertrophic cardiomyopathy.

Page 76: Internal Medicine- Cardiology

Mutation of the CFTR (cystic fibrosis transmembrane conductance regulator) gene (Choice B) causes cstic fibrosis.

Mutations in the gene for cardiac myosin-binding protein C (Choice C) account for approximately 15 percent of cases of familial hypertrophic cardiomyopathy.

Spectrins mutations (Choice E) cause red blood cell and neuronal disorders, e.g., erythroid elliptocytosis and spinocerebellar ataxia.

A mother brings in a 12 month old Caucasian male to the urgent care. He was found eating a flower in the flowerbed this morning. The mother thinks the

flower was foxglove. Afterwards the patient began vomiting and became mildly lethargic, so she bought him in for evaluation. The patient's vitals are T

98.8, P 70 and regular, R17. Physical exam shows a drowsy child only, nontender abdominal exam, cardiac exam with regular rhythm and a rate in the

70s, and is otherwise nonspecific. You suspect digitalis toxicity and order an EKG.

The most likely EKG findings in this case would be which of the following:

1) ventricular tachycardia 2) atrial fibrillation 3) 2:1 AV block 4) sinus rhythm 5) sinus tachycardia

The correct answer is choice C. The patient's heart rate is slower than we would expect for his age, yet the rhythm is regular. Below is an example of how the EKG may appear:

Page 77: Internal Medicine- Cardiology

This patient's presentation is indeed worrisome for digitalis toxicity. The cardiac glycoside medications, such as digoxin, are, in fact, derived from the foxglove plant. Digitalis toxicity may cause symptoms such as dizziness, palpitations, somnolence, confusion, blurred vision, nausea, vomiting, diarrhea, etc. Pupils remain normal and physical exam is often nonspecific. Patients remain afebrile. The EKG manifestations of this condition can vary, including PVC's, varying degrees of heart block, sinus tachycardia, sinus bradycardia, ventricular fibrillation, and asystole. 2:1 AV block is the only one of the answer options that fits with a regular rate of 70 in a 12-month old, however.

Treatment inlcudes supportive care and observation if hemodynamically stable. One should consider activated charcoal for an accidental ingestion, or digitalis-fab antibodies if available. Since the patient has an arrhythmia, one should consider atropine before gastric lavage as a treatment, as lavage may worsen the arrhythmia. Check for electrolyte imbalances and correct them if found. As this patient is in an urgent care center, emergency department transfer may be necessary, depending on his stability. Consider a cardiology consult and/or a poison control center consult.

A 60 year old male with longstanding stable angina presents to the ER with a two day history of escalating substernal chest pain episodes lasting 5-15

minutes and accompanied by shortness of breath, diaphoresis, and palpitations. Although his current complaints are qualitatively identical to his usual

anginal symptoms, they have increased in frequency, severity, and duration. They also seem less responsive to sublingual nitroglycerin than usual.

Page 78: Internal Medicine- Cardiology

Although he has had six episodes during the 24 hours prior to presentation, he has not had chest pain since shortly before his arrival in the ER. His ECG

is normal except for deep T wave inversions in V1-V4 which were not present on an ECG performed 3 months earlier. His vital signs, room air oxygen

saturation, and initial set of cardiac biomarkers are normal.

Which of the following interventions is indicated?

1) admit the patient and begin intravenous digoxin 2) admit the patient and begin enoxaparin or other anticoagulant therapy 3) administer thrombolytic therapy 4) admit the patient for observation while continuing his home medications 5) discharge the patient on increased doses of his regular anti-anginal medication

The correct answer is Choice B. The patient’s presentation is strongly suggestive of an acute coronary syndrome. While he should be monitored for evidence of an evolving ST Elevation MI (STEMI), it is appropriate to initiate treatment for unstable angina/Non-ST Elevation MY (NSTEMI). Regardless of whether the treating physician adopts an initial conservative or invasive strategy, anticoagulant therapy is indicated. Digoxin is a positive inotrope which can have arrhythmogenic side effects. It is not indicated for this patient. Thrombolytic therapy or percutaneous coronary intervention is indicated for patients presenting early in the course of STEMIs, but neither is indicated in the treatment of NSTEMIs. Acute coronary syndromes are associated with significant morbidity and mortality in the short term; this risk may be attenuated by more aggressive intervention than suggested in Choices D and E.

Figure 1: Exam review if contraindications of fibrinolytic therapy

Page 79: Internal Medicine- Cardiology

A 42-year-old woman has noted increasing dyspnea for the past 6 years. On examination rales are auscultated in both lungs. She is afebrile. A chest

radiograph shows an enlarged cardiac silhouette and bilateral pulmonary edema. Past history reveals that, as a child, she suffered recurrent bouts of

pharyngitis with group A beta hemolytic streptococcal.

Which of the following cardiac valves are most likely to be abnormal in this woman?

1) Aortic and tricuspid 2) Mitral and pulmonic

Page 80: Internal Medicine- Cardiology

3) Aortic and pulmonic 4) Tricuspid and pulmonic 5) Mitral and aortic

The correct answer is choice E.

This is a case of rheumatic heart disease complicated with congestive heart failure (enlarged cardiac shadow and pulmonary edema).

Rheumatic heart disease is the most serious complication of acute rheumatic fever (evidenced in this case by the history of recurrent bouts of pharyngitis with group A beta hemolytic streptococci.

In rheumatic heart disease the mitral valve is affected most commonly and severely (65-70% of patients); the aortic valve is affected second most commonly (25%).The tricuspid valve is deformed in only 10% of patients, almost always in association with mitral and aortic lesions, and the pulmonary valve is rarely affected.

Rheumatic fever (RF) is a systemic illness that may occur following group A beta hemolytic streptococcal (GABHS) pharyngitis in children. It is thought to result from an inflammatory autoimmune response. The current incidence of rheumatic fever after GABHS infection is now thought to have decreased to less than 1%.

The proposed pathophysiology for development of rheumatic heart disease is as follows: Cross-reactive antibodies bind to cardiac tissue facilitating infiltration of streptococcal-primed CD4+ T cells, which then trigger an autoimmune reaction releasing inflammatory cytokines (including TNF-alpha and IFN-gamma).

A diagnosis of rheumatic heart disease is made after confirming antecedent rheumatic fever. The modified Jones criteria (revised in 1992) provide guidelines for the diagnosis of rheumatic fever. The Jones criteria require the presence of 2 major or 1 major and 2 minor criteria for the diagnosis of rheumatic fever:

Page 81: Internal Medicine- Cardiology

The major diagnostic criteria include carditis, polyarthritis, chorea, subcutaneous nodules, and erythema marginatum.

The minor diagnostic criteria include fever, arthralgia, prolonged PR interval on ECG, elevated acute phase reactants (increased erythrocyte sedimentation rate [ESR]), presence of C-reactive protein, leukocytosis, and history of beta hemolytic streptococcal pharyngitis.

The attached image is a chest radiograph showing cardiomegaly due to carditis of acute rheumatic fever.

A 75 year-old female has been under treatment for arthritis with four weeks of non-steroidal anti-inflammatory drugs. She now notes two days of

increasing chest pressure with exertion and black, tarry stools. On examination she appears pale, and her vital signs are: Pulse 110/min; BP: 90/60;

Resp: 22; Temp: 98.6 F.

Pathophysiologic mechanisms for her accelerated angina pectoris might include all of the following, except:

1) Increased myocardial oxygen demand due to sinus tachycardia 2) Increased left ventricular wall tension due to increased preload

Page 82: Internal Medicine- Cardiology

3) Increased myocardial contractility due to increased sympathetic tone 4) Decreased coronary perfusion pressure due to hypotension 5) Decreased oxygen carrying capacity due to anemia

The correct answer is Choice B. Myocardial ischemia is defined by insufficient oxygen delivery to meet the metabolic demands of the myocardium. Disease processes associated with myocardial ischemia can therefore be divided into two overlapping etiologic categories: those that cause increased myocardial oxygen consumption and those that cause decreased oxygen delivery. The clinical vignette describes a patient with worsening angina in the context of symptoms and signs suggestive of an upper GI bleed associated with NSAID use. GI bleeds are associated with loss of intravascular volume as well as well as the osmotically active blood components that could otherwise cause compensatory fluid shifts from the extravascular space to the intravascular space. Therefore hemodynamically significant GI bleeds would typically cause decreased rather than increased preload. Blood pressure is determined by the product of cardiac output and peripheral resistance. In order to maintain blood pressure in the setting of a hemodynamically significant loss of intravsacular volume, the cardiovascular system can compensate by increasing peripheral resistance and by increasing cardiac output. Cardiac output, in turn, is determined by the product of the stroke volume and pulse rate. Myocardial oxygen consumption increases in the setting of compensatory mechanisms to increase cardiac output by increasing pulse rate (Choice A) and increasing stroke volume (Choice C). The rate of oxygen delivery supplied by the coronary arteries to the myocardium is dictated by the rate of blood flow and the oxygen content of the blood. As described in the Poiseuille equation, blood flow within an artery will vary directly with the magnitude of the pressure gradient within the artery. The high end of this pressure gradient is determined by the blood pressure in the aortic root, which is decreased in the context of systemic hypotension (Choice D). Although some of the oxygen contained in blood is in solution, blood's oxygen carrying capacity is driven largely by its hemoglobin content, which is reduced following hemodynamically significant GI bleeds (Choice E).

A 37-year-old woman presents to the emergency department with a fever and fatigue. Clinical examination showed a temperature of 38.5C, a pan-

Page 83: Internal Medicine- Cardiology

systolic murmer and fine crackles in both lung fields. A Chest x-ray showed multiple patchy infiltrates . Echocardiography revealed vegetations on the

tricuspid valve and blood cultures grew Staph. Aureus. A diagnosis of acute bacterial endocarditis limited to the tricuspid valve was made.

Which of the following is the most probable etiology?

1) Congenital heart disease 2) intravenous drug use 3) Rheumatic fever 4) Rheumatoid arthritis 5) Systemic lupus erythematosus

The correct answer is choice B

Tricuspid valve endocarditis is mainly a disease of intravenous drug abusers. Virulent pathological organisms, most commonly Staph Aureus, enter via injection sites, then are carried to the right side of the heart and settle on the tricuspid valve. Clinical signs are often absent or minimal. Classical features quoted in textbooks of splinter haemorrhages, Jayne-Way lesions, Roth spots (haemorrhages in the eye) are rarely seen. Affected heart valves, usually the aortic and mitral lead to corresponding murmers.Tricuspid valve endocarditis can be severe and 25% of patients require surgical intervention.

Choice A Congenital tricuspid valve disease and associated endocarditis is very rare.

Choice B Intravenous drug abuse accounts for nearly 50% of cases of tricuspid valve endocarditis

Choice C Rheumatic fever is an acute systemic disease affecting the peri-arteriolar connective tissue and can cause a mild pan-carditis. Chronic rheumatic valvular heart disease occurs with repeated inflammation causing valvular disease.

Choice D Rheumatoid arthritis may rarely cause a non-infective endocarditis.

Page 84: Internal Medicine- Cardiology

Choice E Systeminc lupus erthematosus can be associated with sterile endocarditis, described as Libman-Sacks endocarditis. It is thought to be due to the deposition of immune complexes in the endocardium.

A 64-year-old woman presents to the emergency department with crushing substernal chest pain that lasted for 90 minutes. Physical examination reveals

heart rate of 100 bpm, blood pressure of 75/40 mm Hg, respiratory rate of 22 breaths/min, and jugular venous pressure of 12 cm above the right atrium.

Lungs are clear to auscultation. Cardiac examination reveals normal S1 and S2 and a right-sided S3 with no evidence of mitral or aortic murmur.

What is the most appropriate first step in this patient’s management?

1) Intravenous (IV) nitroglycerin 2) IV dopamine 3) Intra-aortic balloon pump (IABP) 4) Furosemide 5) IV fluid administration

The correct answer is choice E

Page 85: Internal Medicine- Cardiology

This is a classical case of inferior wall myocardial infarction with right ventricular infarction (RVI). ECG shows a 3-mm ST elevation in leads II, III, and aVF, as well as a 1-mm ST elevation in leads V1 and V2. When an occlusion of the right coronary artery occurs, blood flow to the acute marginal and right ventricular branches, which supply the right ventricular free wall, is blocked. If occlusion occurs distal to these marginal branches, RVI does not occur. The diagnosis of RVI should always be considered in patients who have an inferior wall myocardial infarction. Early diagnosis is critical to avoid therapy that may adversely affect the outcome of RVI. The presence of ST-segment elevation in leads II, III, and aVF on an ECG are always suggestive of RVI, plus the classic combination of elevated jugular venous pressure, hypotension, and clear lung fields also suggests RVI, as seen in this case.

Delayed capillary filling, cool clammy skin, hypotension, changes in mental status, decreased urine output, right ventricular gallop, tricuspid regurgitation, and atrioventricular dissociation manifested as 'cannon a waves' in the jugular venous pulse are associated with RVI. However, pericardial tamponade, constrictive pericarditis, and pulmonary embolus may share some of these features and must be excluded. Volume loading with an isotonic solution such as isotonic sodium chloride solution (Choice E) is recommended as the initial therapy in RVI.

Intravenous nitroglycerin choice (A) and furosemide choice (D) are not the correct answers. Nitroglycerin is generally indicated in myocardial infarction because of the drug’s vasodilatory effects on blood vessels, which lead to decreased ischemia, decreased myocardial oxygen consumption, and, subsequently, an improvement in blood flow to the myocardium. However, vasodilators, diuretics, and morphine are not well tolerated by patients with RVI and may lead to severe hypotension. The effects of these medications may cause a reduction in preload by decreasing filling pressures and subsequently decreasing cardiac output and worsen the situation.

IV dopamine choice (B) is not a correct choice as the response to volume loading in patients with RVI varies. If the hemodynamic parameters do not change markedly after volume loading with intravenous fluids, addition of an inotropic medication, such as dobutamine (Dobutrex), may be needed to enhance ventricular contractility and cardiac output. Dobutamine may be effective if volume loading is unsuccessful.

Intra-aortic balloon pump (IABP) (choice C) is not the correct answer. If cardiac output continues to decrease and shock is eminent, an intra-aortic balloon pump can be used to reduce after-load and provide the added benefit of augmentation.

In this case the first step in management of RVI is therefore IV fluid administration.

Page 86: Internal Medicine- Cardiology

A 72-year-old smoker, diabetic male patient with a history of hypercholesterolemia presents to the hospital with recent onset of left sided chest pain

radiating to neck and left arm accompanied by nausea, vomiting and profuse sweating. ECG showed ST-segment elevation in leads II, III, aVF and V4-V6.

All of the following statements about the coronary obstruction causing this patient’s symptoms are true except:

1) There was likely to have been a soft, lipid-laden atherosclerotic plaque recently present at the site of the current obstruction 2) There is likely to be an occlusive thrombus at the site of obstruction 3) There is likely to be a component of coronary vasospasm at or near the site of obstruction 4) There is likely to be increased platelet aggregation at the site of plaque disruption 5) There are evidences that treatment of the patient with aspirin results in reduction in mortality rate

The correct answer is choice C

ST-elevation myocardial infarction (STEMI) generally occurs when coronary blood flow decreases abruptly after a thrombotic occlusion of a coronary artery previously affected by atherosclerosis (choice A). STEMI occur when a coronary artery thrombus (choice B) develops rapidly at a site of vascular injury produced or facilitated by factors such as cigarette smoking, hypertension, and lipid accumulation. Most commonly, infarction occurs due to fissuring, rupture or ulceration of atherosclerotic plaque and when conditions favor thrombogenesis (Choice D), a mural thrombus forms at the site of rupture leading to coronary artery occlusion.

Rarely STEMI may occur due to coronary artery occlusion caused by coronary emboli, congenital abnormalities, coronary spasm (Choice C), and other systemic diseases.

Aspirin is essential in the management of patients with suspected STEMI and is effective across the entire spectrum of acute coronary syndromes. The evidence for the benefits of antiplatelet therapy (mainly with aspirin) in STEMI has been shown in the comprehensive overview by the Antiplatelet Trialists’ Collaboration in which data from randomized trials revealed a relative reduction of 27% in the mortality rate in patients receiving antiplatelet agents (Choice E).

The statement there is likely to be a component of coronary vasospasm at the site of obstruction (Choice C) is not true as coronary vasospsm is a rare cause of STEMI.

Page 87: Internal Medicine- Cardiology

A 17 year old high school senior complains of with amenorrhea for the past four months. Previously, her menses had occurred at regular 28 day intervals

since menarche at age 11. The girl admits having been under a great deal of stress with college applications and extracurricular activities such as

cheerleading. Upon questioning, she acknowledges eating large amounts of food such as an entire large pizza when she is feeling stressed. She feels

guilty after she eats, however, and forces herself to vomit it all up. The rest of the time, she eats a ―normal diet‖ of salads and granola.

Physical examination reveals an extremely thin female, with height is 69 in. (175 cm), weight is 101 lbs. (46 kg). There is diffuse fine hair growth on the

extremities and mild erosion of the enamel of the anterior teeth. A callus is present on the dorsal surface of the index and middle fingers of the patient’s

right hand. TSH is within normal limits. A urine pregnancy screen is negative.

Which of the following is the most appropriate diagnosis in this patient?

1) Anorexia nervosa 2) Obsessive-compulsive disorder 3) Borderline personality disorder 4) Bulimia nervosa 5) Hyperprolactinemia

The correct answer is choice A.

The criteria for anorexia nervosa include amenorrhea, body weight less than 85% of expected ideal weight, and intense fear of gaining weight or becoming fat. This patient fits those criteria, and although binging and purging behaviors are more typically associated with bulimia, her disease is more appropriately accounted for by anorexia nervosa (choice A).

Put another way, anorexia nervosa is the supraordinate diagnosis in the DSM-IV – that is, a patient who seems to meet criteria for bulimia but who also meets criteria for anorexia should be diagnosed with anorexia. The reason for this is prognostic: anorexia nervosa has a staggering 20% mortality rate at 20 years. Thus, the most appropriate diagnosis in this patient is anorexia nervosa, binge-eating/purging type.

As a sidenote, patients with purging behaviors do have a couple of characteristic signs on physical exam, and these are often important clues on standardized medical exams. Erosion of the enamel of the anterior teeth occurs due to exposure of the teeth to acidic gastric juices. There may also be enlarged parotid glands, or a positive Russell sign (described in the question stem as the callus on the patient’s dominant hand due to self-induced

Page 88: Internal Medicine- Cardiology

vomiting). The presence of lanugo, or fine downy hair, can be seen in any underweight patient.

Patients with anorexia may have comorbid obsessive-compulsive personality disorder or even full-blown obsessive compulsive disorder (choice B), but there is nothing in the question stem that suggests either obsessions (persistent and intrusive ideas or images) or compulsions (impulses for repetitive or stereotyped behavior).

Patients with binging and purging behaviors often have comorbid borderline personality disorder (choice C). This is a commonly tested disorder on standardized exams, but none of the classic features are given to you in the question stem such as self injury or mutilation, a history of suicidal gestures, or ―splitting.‖

As described above, although purging behaviors are commonly associated with bulimia nervosa (choice D), this patient is underweight and amenorrheic, so the best diagnosis is anorexia nervosa.

Hyperprolactinemia (choice E) can cause secondary amenorrhea, and after a negative pregnancy test and a normal TSH, measuring prolactin is the usual next step in diagnosing amenorrhea. Here, though, the cause of the patient’s amenorrhea is clear – it’s her eating disorder.

A 23-year-old man with rheumatic mitral regurgitation presents with frequent palpitations and exercise intolerance.

Which one of the following is not an indicator of the severity of his mitral regurgitation?

1) Third heart sound 2) The loudness of the murmur 3) Signs of heart failure 4) Presence of thrill 5) S4

The correct answer is Choice E

Page 89: Internal Medicine- Cardiology

Mitral valve regurgitation occurs when there is valvular insufficiency and blood regurgitates from the left ventricle into the left atrium. The most common cause is mitral valve prolapse, rheumatic heart disease is becoming less common. A third heart sound (Choice A) occurs when the left ventricle becomes dilated or fails and is related to rapid filling. It therefore is common in more severe cases of mitral valve regurgitation. A "gallop" rythm can be heard in patients with tachycardia and left ventricular failure.

The loudness of the murmur (Choice B) is proportionate to the degree of regurgitation.

Signs of heart failure (Choice C) occur when mitral valve regurgitation is more advanced and often with long standing cases.

The presence of a thrill (Choice D) is related to the loudness of the murmer and therefore the extent of mitral valve regurgitation.

The fourth heart sound (Choice E) is associated with increased stiffness of the ventricle and causes include: hypertrophy, overloading or fibrosis of the ventricle, long-standing hypertension, aortic stenosis and Congestive Heart Failure. It is not associated with mitral regurgitation.

A 58-year-old chronic alcoholic man has had an enlarging abdomen for 5 months. He has experienced no abdominal or chest pain. On physical

examination he has a non-tender abdomen with no masses palpable, but there is a fluid wave. An abdominal CT scan shows a large abdominal fluid

collection with a small cirrhotic liver. A chest radiograph shows a globally enlarged heart. He has vital signs showing T 37.1 C, P 78/minute, RR

16/minute, and BP 115/75 mm Hg.

Which of the following cardiovascular conditions is he most likely to have?

1) Severe occlusive coronary atherosclerosis 2) Hypertrophic cardiomyopathy 3) Restrictive cardiomyopathy 4) Myocardial amyloid deposition 5) Dilated cardiomyopathy

Page 90: Internal Medicine- Cardiology

The correct answer is choice E.

This is a case of alcoholic dilated cardiomyopathy and alcoholic liver cirrhosis.

The mechanism of the cardiac damage produced by alcohol remains unclear. Recent theories indicates a direct toxic effect of ethanol or its metabolites on the cardiac muscle. Original theories regarding the mechanism focused on nutritional deficiencies (eg, thiamine deficiency), secondary exposures (eg, tobacco, cobalt, arsenic), and other comorbidities (eg, hypertension).

In such cases there is interstitial fibrosis of the cardiac muscle with degeneration of some myocytes and hypertrophy of other myocytes. This leads to 4-chamber dilatation and overall increased cardiac mass with progressive weakness of the cardiac muscle leading to dilated cardiomyopathy. Alcoholic heart disease is pathologically indistinguishable from other dilated cardiomyopathy. Diagnosis depends on a history of excessive alcohol consumption and the absence of any other known cause of cardiomyopathy.

Usually, alcoholism leads to hypertension but in this case there is relative hypotension due to diminished cardiac output.

Alcoholic cirrhosis is a condition of irreversible liver disease due to the chronic inflammatory and toxic effects of ethanol on the liver. Alcoholic cirrhosis develops for between 10% and 20% of individuals who drink heavily for a decade or more as little as 3-4 drinks a day in some men and 2-3 in some women. In this case there is cirrhosis complicated with ascites leading to the abdominal enlargement and the fluid wave.

Severe occlusive coronary atherosclerosis (choice A) would lead to ischemic chest pain and there would not be this obvious cardiac enlargement.

Hypertrophic cardiomyopathy (choice B) is usually presented with syncopal attacks, dyspnea (RR is normal in this case), anginal pain and palpitation which are missing in this case. Also the cardiac usually is on the left side only.

Page 91: Internal Medicine- Cardiology

In restrictive cardiomyopathy (choice C) There would be cardiomegaly but there also would be manifestations of systemic congestion due to diminished ventricular compliance. These manifestations include congestive hepaomegaly (not cirrhosis), pulmonary congestion leading to dyspnea and paroxysmal nocturnal dyspnea orthopnea, pripheral edema and ascites.

Myocardial amyloid deposition (choice D) is the most common cause of restrictive cardiomyopathy in adults. There would be manifestations of systemic congestion as well.

Which one of the following indicates an absolute contraindication to thrombolytic therapy in acute ST segment elevation myocardial infarction? 1) A 70-year old hypertensive male presenting as an inferior wall MI with suspicion of having aortic dissection 2) A 40-year old woman having 24-weeks pregnancy 3) A 55-year old diabetic male with proliferative diabetic retinopathy with an anterior wall MI 4) A 68-year old hypertensive male patient with a blood pressure of 180/100, which is then easily controlled with antihypertensive medications

showing ST segment elevation in lead V1, V2, and V3 5) A 60-year old male who is a known case of ischemic heart disease presents with a new left bundle branch block and chest pain for 2 hours

with ST segment elevation in V4, V5, and V6

The correct answer is choice A

The following table shows the contraindication and cautions for thrombolysis in patients of ST-segment elevation myocardial infarction.

Page 92: Internal Medicine- Cardiology

Reperfusion Therapy with Fibrinolysis (see reference below) For patients with ischemic symptoms characteristic of acute myocardial infarction (MI) of < 12 hours duration and persistent ST-segment elevation (STE), guidelines recommend that all undergo rapid evaluation for reperfusion (primary percutaneous coronary intervention [PCI] or fibrinolytic) therapy and have a reperfusion strategy implemented promptly after contact with the health-care system (Grade 1A).

1. In patients with acute MI who are candidates for fibrinolytic therapy, the guideline developers recommend administration as soon as possible (ideally within 30 min) after arrival to the hospital or first contact with the health-care system (Grade 1A).

2. In health-care settings where prehospital administration of fibrinolytic therapy is feasible, the guideline developers recommend prehospital administration of fibrinolytic therapy (Grade 1A).

Page 93: Internal Medicine- Cardiology

3. For patients with ischemic symptoms characteristic of acute MI of < 12 hours duration, and persistent STE, the guideline developers recommend administration of streptokinase, anistreplase, alteplase, reteplase, or tenecteplase over no fibrinolytic therapy (all Grade 1A).

4. For patients with symptom duration < 6 hours, the guideline developers recommend the administration of alteplase (Grade 1A) or tenecteplase (Grade 1A), and suggest reteplase (Grade 2B) over streptokinase.

5. For patients receiving fibrinolytic therapy, the guideline developers suggest the use of a bolus agent (e.g., tenecteplase) to facilitate the ease of administration and potentially reduce the risk of nonintracranial hemorrhage (ICH)-related bleeding (tenecteplase) (Grade 2A).

6. For patients with ischemic symptoms characteristic of acute MI of < 12 hours duration, and left bundle-branch block (BBB) with associated STE changes, the guideline developers recommend fibrinolytic therapy if primary percutaneous coronary intervention (PCI) is not readily available (Grade 1B).

7. For patients with ischemic symptoms characteristic of acute MI of < 12 hours duration and ECG findings consistent with a true posterior MI, the guideline developers suggest fibrinolytic therapy if primary PCI is not readily available (Grade 2B).

8. For high-risk patients with ongoing symptoms characteristic of acute MI or hemodynamic compromise and duration of 12 to 24 hours who have persistent STE or left BBB with STE changes, the guideline developers suggest fibrinolytic therapy if primary PCI is not readily available (Grade 2B).

9. In patients with any history of intracranial hemorrhage (ICH), or with history of head trauma, or with ischemic stroke within the past 6 months, the guideline developers recommend against administration of fibrinolytic therapy (Grade 1C).

A 72-year old male presents to cardiac out-patient department for a routine examination. He is asymptomatic at present. He has a past medical history of hypertension and hypercholesterolemia. He is also a chronic smoker. He is taking hydrochlorthiazide 25 mg daily, atenolol 25 mg daily, and simvastatin 40 mg nightly. On examination, pulse rate is 70 per minute and blood pressure is 130/80. On auscultation, bilaterally vesicular breath sounds are heard.

Page 94: Internal Medicine- Cardiology

Cardiac auscultation is unremarkable. During abdominal examination, a pulsatile mass is felt just left to umbilicus. CT scan of abdomen confirms a diagnosis of abdominal aortic aneurysm. Which of the following statements is not correct regarding the role of different imaging modalities in establishing the diagnosis of his condition?

1) Abdominal ultrasonography can visualize an aneurysm in the transverse and longitudinal planes and can accurately define aneurysm size to within ±0.3 cm

2) Among all the imaging modalities, abdominal ultrasonography gives most reliable measurement of suprarenal aorta 3) Computed tomography is extremely accurate in both diagnosing and sizing aortic aneurysms to within ±0.2 cm. and is better than

ultrasonography in defining the shape and extent of the aneurysm as well as the anatomical relation of the visceral and renal vessels 4) MRA is extremely accurate in determining aneurysm size, and it correctly defines the proximal extent of disease 5) Aortography is excellent in defining the suprarenal extent of the aneurysm and associated renal, mesenteric, or iliofemoral arterial disease

The correct answer is choice B Several diagnostic imaging modalities are currently used for detecting, sizing, and serial monitoring of abdominal aortic aneurysms and for precise definition of the aortic anatomy preoperatively. Abdominal ultrasonography can visualize an aneurysm in the transverse and longitudinal planes, with a sensitivity of 87 to 99 percent and can accurately define aneurysm size to within ±0.3 cm, but it is limited by less reliable measurements of the suprarenal aorta and significant interobserver variability. Its major advantages are that it is relatively inexpensive, is noninvasive, and does not require the use of a contrast agent. Computed tomography is an extremely accurate method for both diagnosing aortic aneurysms and sizing them to within ±0.2 cm. CT has an advantage over ultrasonography in that it can better define the shape and extent of the aneurysm as well as the local anatomical relation of the visceral and renal vessels. Its disadvantages are that the procedure is more expensive and less widely available than ultrasonography, and it also requires the use of ionizing radiation and intravenous contrast. Although CT may be less practical than ultrasonography as a screening tool, its high accuracy in sizing aneurysms makes it an excellent modality for serially monitoring changes in aneurysm size. Spiral CT provides more comprehensive evaluation of the anatomy of an abdominal aortic aneurysm and information regarding renal, mesenteric, or iliac arterial occlusive disease which is usually sufficient for preoperative evaluation of abdominal aneurysms. Magnetic resonance angiography (MRA) is also an alternative for the preoperative evaluation of aortic aneurysms. Tomographic images are reconstructed to create a three-dimensional image; the aorta can be visualized from a series of projections to facilitate appreciation of anatomical relationships. MRA is extremely accurate in determining aneurysm size, and it correctly defines the proximal extent of disease and iliofemoral

Page 95: Internal Medicine- Cardiology

involvement in more than 80 percent of cases. Aortography may underestimate aneurysm size in the presence of nonopacified mural thrombus lining the aneurysm walls, but it remains an excellent technique for defining the suprarenal extent of the aneurysm and any associated renal, mesenteric, or iliofemoral arterial disease. Its disadvantages are that it is expensive, it is an invasive procedure with inherent risks, and it requires the use of intraarterial contrast and ionizing radiation. Preoperative aortography is now used only in selected cases because CT and MRA provide sufficient information in most cases.

A 35-year-old previously healthy woman dies suddenly and unexpectedly. At autopsy, one of her cardiac valves demonstrates attenuation of the chordae

tendineae, with rupture of one of the chordae. On microscopic examination, one of the mitral leaflets show myxomatous change.

Which of the following is the most likely cause for her death?

1) Carcinoid heart syndrome 2) Mitral valve prolapse 3) Rheumatic heart disease 4) Infective endocarditis 5) Acute myocardial infarction

The correct answer is choice B.

Mitral valve prolapse (MVP), the most common anomaly of the mitral valve apparatus. It occurs when one or both mitral valve leaflets excessively billows into the left atrium toward the end of systole. This prolapse may be isolated (as in this case), in association with generalized connective tissue abnormalities as Ehlers-Danlos syndrome or in the context of inflammatory disease such as rheumatic fever or endocarditis.

Urokinase-plasminogen activator (PLAU) is thought to produce myxomatous degeneration of collagen fibrils of the valve leaflets leading to excess mucopolysaccharides in the middle spongiosa layer of the mitral valve leaflets. Degeneration of collagen within the central core of the chordae tendineae may lead to chordal rupture (both features are seen in the autopsy of this case).

Clinically, this disease may be presented by chest pain due to excessive stretching of the chordae tendineae leading to traction on papillary muscles, palpitation due to cardiac arrhythmia, fatigability and dyspnea and there may be associated skeletal abnormalities such as hypomastia, arachnodactyly, scoliosis, crowding of teeth and joint hypermobility. On auscultation, apical midsystolic nonejection click and late systolic murmur are heard. Heart sounds

Page 96: Internal Medicine- Cardiology

are usually normal.

It may be complicated by mitral regurge, congestive heart failure, infective endocarditis or arrhythmia. The most serious complication is sudden cardiac death (as in this case) which may be due to rupture of chordae tendineae or more probably due to excessive atrial or ventricular arrhythmias.

The associated image is a chest radiograph of a case with mitral valve prolapse and mild mitral regurgitation. The radiograph shows cardiomegaly and normal pulmonary vasculature.

Page 97: Internal Medicine- Cardiology

Carcinoid heart syndrome (choice A) is the cardiac manifestations of the carcinoid tumor. Carcinoid tumors are of neuroendocrine origin and derived from primitive stem cells in the gut wall. They release many active substances as tumor growth factor (TGF)-beta, beta–fibroblast growth factor (beta-FGF), and platelet-derived growth factor. In heart, these substances lead to fibrosis of the endocardium, which often involves the right side of the heart including

Page 98: Internal Medicine- Cardiology

ventricular aspect of the tricuspid valve and associated chordae.

In rheumatic heart disease (choice C) the pathological findings would be vegetations and fibrosis of the affected valves.

In infective endocarditis (choice D) there would be sterile fibrin-platelet vegetations or thrombi on the affected valve and chordae. Abscesses may be present on the endocardium and extending to the myocardium.

In acute myocardial infarction (choice E) there would be a thrombus that obstructs an atherosclerotic coronary artery. The pathology of the cardiac muscle depends on the duration since the infarction was established:

Before 6 to 12 hours: No visible lesion is seen (Acute cases)

By 18 to 24 hours: Infarct area becomes pale to cyanotic

In the first week: The infarct area becomes progressively more sharply defined, yellow and softened

By the 7 to 10 days, circumference of the infarct area becomes hyperemic, and progressively expands

By the 6 weeks, fibrous scar is well established

Page 99: Internal Medicine- Cardiology

A 30-year-old female presents with progressive exercise induced breathlessness and fatigue. Clinical examination revealed a pan systolic murmer, a loud

and split second heart sound and a raised jugular venous pressure.There was no evidence of lung disease, an echocardiogram showed right ventricular

dilatation and tricuspid regurgitation. A diagnosis of primary pulmonary hypertension was made following right sided heart catheterisation with pressure

monitoring.

In primary pulmonary hypertension, all of the followings are true, except?

1) An association with HIV infection has been documented 2) Vascular intimal and medial (smooth muscle) proliferation develops leading ultimately to thrombotic pulmonary arteriopathy 3) Large V waves are often present 4) Primary medical treatment is curative 5) Most patients die within 2-3 years of diagnosis

The correct answer is Choice D

Pulmonary hypertension is an increase in blood pressure in the lung vasculature, leading to breathlessness, dizziness, fainting, all exacerbated by exertion. It is not associated with breathlessness when lying flat compared with left sided heart failure.

Diagnosis is definitively made with right sided heart catheterisation and insertion of a Swan-Ganz catheter to directly measure venous pressure. Pulmonary hypertension is present when mean pulmonary artery pressure exceeds 25 mm Hg (3300 Pa) at rest or 30 mm Hg (4000 Pa) with exercise.

Causes of pulmonary hypertension include:

Familial

Page 100: Internal Medicine- Cardiology

Idiopathic (Primary)

Collagen vascular disorders, HIV, toxins and drugs

Venous or capillary disease

Left sided heart disease

Chronic pulmonary disease (most common cause seen in clinical practice)

Chronic thrombotic or embolic disease

Choice A HIV has been documented as a rare cause of pulmonary hypertension and its cause is not known

Choice B The pathophysiological process in primary pulmonary hypertension consists of vascular intimal and medial (smooth muscle) proliferation and with progression, a thrombotic pulmonary arteriopathy develops

Choice C The physical signs of primary pulmonary hypertension often reveals the following findings:

Page 101: Internal Medicine- Cardiology

The pulmonary component of the second heart sound is usually increased,

Pulmonary valve regurgitation may occur.

A murmur of tricuspid regurgitation can be present.

Jugular venous pressure may be elevated. Large V waves are often present because of severe tricuspid regurgitation

Choice D There is no cure for primary pulmonary hypertension.

Choice E The typical 3 year survival rate of patients with primary pulmonary hypertension is around 30%. The recent use of prostacyclin analogues, endothelin receptor antagonists, phosphodiesterase-5 inhibitors, has however, improved the prognosis.

A 57-year-old man is brought to the emergency room with a 2 hour history of central crushing chest pain radiating to the left arm. Clinical examination

revealed a sweaty distressed gentleman with a pulse rate of 130, blood pressure of 140/95, but no other signs of cardiac or respiratory disease.

An electrocardiogram showed a wide-complex ventricular tachycardia at a rate of 126 beats per minute. The physician prescribed a drug to decrease

Sino atrial (SA) node automaticity, increase Atrio Ventricular (AV) node refractoriness, and decrease AV node conduction velocity.

Which of the following agents was most likely prescribed?

1) Amiodarone 2) Disopyramide 3) Lidocaine 4) Propranolol 5) Verapamil

Page 102: Internal Medicine- Cardiology

The correct answer is choice D

Ventricular tachycardia is a serious arrythmia of the ventricles that can cause significant morbidity and mortailty. It is most commonly associated with coronary heart disease (including myocardial infarction), but also occurs in myopathies and valvular heart disease. The ECG findings are of a broad complex tachycardia with a rate greater than 100 beats per minute. It can cause chest pain, sweating and hypotension.

Immediate treatment will include CPR if there is cardiac arrest, electrical defibrillation or cardioversion for severely compromised patients and anti-arrhythmic medications, such as lidocaine, procainamide, sotalol, or amiodarone) intravenously. In this particular patient, the latter option would be the most appropriate. Beta blockers are usually not the first line treatment, but is the corrrect answer defined by the mechanism of action (see below).

Choice A Amioderone belongs to the class III anti-arrhythmics, with weak class I, II and class IV actions. It prolongs the effective refractory period of myocardial cells, AV node and anomalous pathways. It also depresses automaticity of SA node and AV node.

Choice B Disopyramide, a class Ia antiarrhythmic, has similar action to quinidine (sodium channel blocker). It acts by decreasing myocardial excitability and conduction velocity. It lengthens the effective refractory period of the atrium. It is used in the treatment of ventricular tachycardias, but its anti muscarinic effects preclude its use as a drug of first choice.

Choice C Lidocaine, a class 1b local anaesthetic, is a membrane stabilizing drug; it reversibly decrease the rate of depolarization and repolarization of excitable membranes.

Choice D Propanolol, a class III beta-blocker, prolongs action potential duration in atria, ventricles, AV node and accessory AV pathways.

Choice E Verapamil is a calcium channel blocker and belongs to class IV antiarrhythmic agents. Since calcium channels are especially concentrated in the SA and AV nodes, verapamil can be used to decrease AV node conduction

Page 103: Internal Medicine- Cardiology

A 17-year-old girl "blacks out" while out running for exercise one afternoon, as she has done for many years. She is taken to the emergency room, where

a physical examination, chest radiograph, head CT scan, CBC, and chemistry panel are all normal. Over the next year, she develops mild dyspnea and

fatigue. She experiences several episodes of near-syncope.

After another syncopal episode, she is referred to a cardiologist who orders an EKG that shows changes of left ventricular hypertrophy and broad Q

waves. An echocardiogram reveals left ventricular and septal hypertrophy, small left ventricle, and reduced septal excursion. The septum has a "ground

glass" appearance. She then dies suddenly and unexpectedly. The microscopic appearance of the septum with trichrome stain reveals myofiber disarray.

Which of the following is the most likely diagnosis?

1) Rheumatic heart disease 2) Viral myocarditis 3) Acute pericarditis 4) Hypertrophic cardiomyopathy 5) Dilated cardiomyopathy

The correct answer is choice D.

Hypertrophic cardiomyopathy (HCM) is a genetic disorder that is typically inherited in an autosomal dominant fashion with variable penetrance and variable expressivity. It results from defects in sarcomeric proteins of the cardiac myocyte. These defects result in myofibril disarray and fibrosis that progresses over time and contributes to ventricular hypertrophy. Hypertrophy frequently involves the interventricular septum which can result in outflow tract obstruction. Although the ventricle hypertrophies, the ventricular cavity itself does not dilate so the systolic function remains intact and the diastolic one is markedly affected. In late stages of the disease, patients may progress to heart failure with ventricular dilatation.

It is usually presented with syncopal attacks, dyspnea and easy fatigability due to diminished diastolic function decreasing the cardiac output. It may be complicated by arrhythmia leading to palpitations.Myocardial ischemia (wide Q wave) arises due to impaired diastolic relaxation and markedly increased myocardial oxygen consumption due to ventricular hypertrophy. Congestive heart failure may occur as a result of a combination of impaired diastolic function and myocardial ischemia.

The most serious complication is sudden cardiac death. It is caused by ventricular fibrillation which occurs in more than 80% of individuals with

Page 104: Internal Medicine- Cardiology

hypertrophic cardiomyopathy.

The associated image shows hypertrophic cardiomyopathy. Oblique magnetic resonance angiogram (outflow 2-chamber view equivalent to a long-axis echocardiogram) shows an area of signal intensity loss (white arrow) in the left ventricular outflow tract, where an obstruction between the hypertrophied septum and anterior mitral leaflet is present. The obstruction is well below the aortic valve ring (between black arrows).

In rheumatic heart disease (choice A) there would be a classic history of recurrent bouts of pharyngitis followed by acute attack of rheumatic fever. To

Page 105: Internal Medicine- Cardiology

diagnose a case of rheumatic heart disease we should look for modified Jones criteria which we are lacking in this case. The Jones criteria require the presence of 2 major or 1 major and 2 minor criteria for the diagnosis of rheumatic fever:

The major diagnostic criteria include carditis, polyarthritis, chorea, subcutaneous nodules, and erythema marginatum.

The minor diagnostic criteria include fever, arthralgia, prolonged PR interval on ECG, elevated acute phase reactants (increased erythrocyte sedimentation rate [ESR]), presence of C-reactive protein, leukocytosis, and history of beta hemolytic streptococcal pharyngitis.

Viral myocarditis (choice B) may be presented with heart failure but it would not produce the left ventricular and septal hypertrophy and themicroscopic appearance of the septum seen in this case. Also its prognosis is very good and don’t produce sudden cardiac death.

Acute pericarditis (choice C) is mainly presented by dull aching or sharp chest pain with pericardial friction rub which are pathogonomonic for pericarditis. No ventricular hypertrophy or broad Q wave shall be present in acute cases.

In dilated cardiomyopathy (choice E), echocardiogram would show dilatation of the left ventricle with thinning of its wall.

A 50 year old man is admitted to the hospital within 2 hours of the onset of nausea, vomiting, and acute crushing agonizing left sided chest pain. He has a

family history of early coronary artery disease. The pain was not relieved with sublingual nitroglycerin. An ECG shows ST-segment elevation in leads Ι,

aVL and V3-V6. Findings on physical examination include: pulse rate 100 per minute, regular; Blood pressure 130/90 mm Hg; chest auscultation showing

bilaterally vesicular breath sounds present, no adventitious sound was heard, other findings were also unremarkable.

Which of the following is the most appropriate management to decrease myocardial damage and mortality in this case?

1) Administration of digitalis 2) Primary PCI 3) Admistration of quinidine

Page 106: Internal Medicine- Cardiology

4) Coronary artery bypass grafting withing 1 week 5) Thrombolytic therapy

The correct answer is choice E

The symptoms and ECG findings in this patient suggest a diagnosis of acute lateral wall myocardial infarction. The most important management strategy at this stage is to consider the patient for reperfusion therapy. The goal of reperfusion is to restore blood flow to the ischemic myocardium and prevent ongoing cell death. Therefore, the method by which reperfusion can be achieved most quickly should be used. Primary PCI has been shown to result in improved patency rates of the infarct-related artery, compared with fibrinolysis. With recent advanced techniques, the outcome of coronary stenting has improved compared to balloon angioplasty alone.

Digitalis is used for treatment of congestive heart failure. Digitalis (choice A) is no role at this stage as there are no findings which are suggestive of heart failure.

Quinidine (choice C) is an antiarrhythmic drug used to treat arrhythmias. At this stage, quinidine is not indicated in this patient as there are no evidences of arrhythmia in ECG. However, very early after the onset of symptoms, the thrombus in the infarcted artery is still soft and may be recanalized by thrombolysis as quickly as primary PCI. The thrombus may remain soft for the first 3 hours after onset of symptoms. Therefore, fibrinolysis (choice E) is an effective treatment within 3 hours of symptom onset. After 3 hours, primary PCI (choice B) offers clear-cut benefit over fibrinolysis and is considered the preferred option.

Coronary artery bypass grafting (choice D) performed within one week is not beneficial in decreasing myocardial damage which is going on in this patient having acute myocardial infarction. Decisions about doing coronary artery bypass grafting may be taken after coronary angiogram.

Patients should be instructed to use antibiotic prophylaxis for infective endocarditis before dental procedure when they have which of the following conditions?

1) Completely repaired congenital heart disease (8 months post-op) 2) A small muscular ventricular septal defect

Page 107: Internal Medicine- Cardiology

3) Previous resection of an abdominal aortic aneurysm 4) Completely repaired congenital heart disease (>1 year post-op) 5) Cardiac transplant recipients with or without cardiac valvulopathy

The correct answer is choice B

The following table shows recommendation of American Heart Association for the prevention of infective endocarditis.

The subendocardium of the heart is the most vulnerable to ischemia. Irreversible ischemic injury occurs after coronary artery occlusion, first in the

subendocardium and progressing toward the subepicardium over time.

Page 108: Internal Medicine- Cardiology

Which of the following statement correctly represents the major reason for this:

1) When cardiac work load increases subendocardial blood flow depends on left ventricular systolic pressure and duration of systole 2) Blood flow to the subendocardium is almost completely diastolic whereas other regions are perfused during systole as well 3) The subendocardial muscles are smallest in the ventricular wall and have a diminished aerobic capacity 4) The subendocardium gets oxygenated from the blood in the ventricular cavity 5) The force of contraction in subenocardial fibres is less as shown by their shorter length in diastole than the superficial fibres

The correct answer is choice B

The branches of coronary arteries arise from the aortic root and are distributed on the epicardial surface of the heart. This gives rise to intramural branches that supply the cardiac muscle. Myocardial ischemia generally appears first and is more extensive in the sub-endocardial region as this is the deepest layer and farthest from the blood supply. When the left ventricle contracts it compresses vessels in its wall and reduces coronary flow. The compressive force is greatest in subendocardial muscle. As a result, in systole no blood flows through subendocardial muscle, whereas the outer subepicardial muscle is perfused. However, all the muscle is perfused in diastole. The subendocardial flow is entirely diastolic and hence subendocardium depends upon diastolic perfusion to maintain its blood supply. The dependence of subendocardium on diastolic blood supply (choice B) and its higher energy requirement makes it more vulnerable to ischemia and infarction. The subendocardial muscles are the largest in the ventricular wall. In electron microscopic studies sarcomeres appear longer in diastole and shorter in systole than superficial fibres (choice E). When the work load on ventricle increases it needs more oxygen and thus more coronary blood flow. The coronary vessels dilate and vascular resistance increases to allow increased blood flow independent of perfusion pressure. However, when cardiac work increases until coronary vessels dilate maximally then coronary flow will depend upon the perfusing pressure. When this happens, subendocardial flow (which is entirely diastolic) will depend on the following

Page 109: Internal Medicine- Cardiology

Coronary arterial diastolic pressure

Left ventricular diastolic pressure

Duration of diastole (choice A)

An 82 year-old woman presents to the Emergency Room with a history of new-onset syncope. Upon arrival, she is awake and oriented and has a small

laceration on her forehead. Her blood pressure is 165/86 mmHg and her heart rate is 43 beats /minute. She is receiving unknown medications for chest

discomfort provoked by exertion and relieved by rest.

Which of the following combinations of drugs would most likely explain this patient’s presentation?

1) Diltiazem and Nitroglycerin spray 2) Verapamil and Atenolol 3) Nifedipine and Isosorbide mononitrate 4) Amlodipine and Nitroglycerin sublingual 5) Sildenafil and Isosorbide dinitrate

The correct answer is Choice B

The medical history (chest discomfort provoked by exertion and relieved by rest) defines that the patient is suffering from angina pectoris. There are currently three major classes of antiischemic drugs used in the medical management of angina pectoris: nitrates (short and long acting), beta blockers and calcium channel blockers. Addition of a second drug is indicated if the angina persists with monotherapy. Since beta blockers and calcium channel blockers appear to be of equivalent efficacy, the choice of one or another as initial therapy is usually based upon other factors, such as associated medical conditions.

Page 110: Internal Medicine- Cardiology

Sublingual nitroglycerin remains the treatment of choice for an acute anginal attack or for prophylaxis prior to activities known to exacerbate angina. In case of chronic therapy for stable angina nitrates are usually reserve as second line antianginal medications because of the problems with tolerance and rebound angina in the nitrate-free interval. In our case the patient’s symptoms (new-onset syncope, low heart rate and normal blood pressure) most likely developed due to bradycardia and transitory heart block. These are common side effects that can occur in case two negative chronotropic drugs, such as atenolol and verapamil, are prescribed together. Avoid prescribing non-dihydropyridine calcium channel blockers such as verapamil or diltiazem to the patients who are on beta-blockers therapy (high risk of heart block)! Nitrates (nitroglycerin sublingual/nitroglycerin patch/isosorbide dinitrate/isosorbide mononitrate,) can cause adverse effects such as orthostatic hypotension along with reflex tachycardia, especially in dehydrated patients. Avoid combined use of nitrates with sildenafil or tadalafil because the combination can cause severe hypotension. In our case the patient has bradycardia and normal blood pressure. Besides, sildenafil is used to treat pulmonary arterial hypertension and erectile dysfunction (Choices A, D, E). Short-acting dihydropyridines, especially nifedipine, should be avoided in treatment of stable angina because it can cause uncontrollable hypotension and there are evidences of an increase risk of acute myocardial infarction in hypertensive patients (Choice C).

While playing cards, a 63-year-old woman has the sudden onset of "knife-like" pain in the chest radiating to the back. She has been previously healthy except for a history of poorly controlled hypertension. Paramedics are called, and she is transported to the hospital. On admission, she has a heart rate of 90/minute, respirations 20/minute, temperature 36.8 C, and blood pressure 150/100 mm Hg. No murmurs, rubs, or gallops are audible. A chest radiograph reveals a widened mediastinum. Laboratory findings include a total serum creatine kinase of 55 U/L, creatinine 0.9 mg/dL, and glucose 123 mg/dL. Which of the following is the most likely diagnosis?

1) Fibrinous pericarditis 2) aortic dissection 3) Infective endocarditis 4) Dilated cardiomyopathy 5) Myocardial infarction

Page 111: Internal Medicine- Cardiology

The correct answer is choice B.

Aortic dissection is a longitudinal split in the media of the aorta. An intimal tear connects the media with the aortic lumen. This tear in the intimal layer result in the propagation of dissection (proximally or distally) secondary to the pressure of blood entering the intima-media space resulting in an exit tear. So a true and a false lumen are created. The smaller true lumen is lined by intima, and the false lumen is lined by media. Most classic aortic dissections begin at 3 distinct anatomic locations: the aortic root; 2 cm above the aortic root; and just distal to the left subclavian artery.

The pathogenesis of aortic dissection is not well understood, but the following are associated risk factors:

Chronic hypertension, 70% have elevated blood pressure

Aortic aneurysm

Connective tissue disorders (Marfan syndrome, Ehlers-Danlos syndrome)

Bicuspid aortic valve

Coarctation of the aorta

It is usually presented by ripping or tearing pain in the intrascapular area with abrupt onset, altered mental status, symptoms of cerebrovascular accident, syncope, limb paresthesia, horner syndrome, dyspnea, dysphagia, flank pain if the renal arteries are involved, hypertension or hypotension if associated

Page 112: Internal Medicine- Cardiology

with cardiac tamponade, hypovolemia, and excessive vagal tone.

The attached image is a plain anteroposterior view of the chest demonstrates a wide mediastinum.

Fibrinous pericarditis (choice A) is false as a pericardial friction rub is pathogonomonic for acute pericarditis and also fever is a hallmark of the disease.

Infective endocarditis (choice C) is false as the patient is not feverish. Also the vast majority of patients (99% of cases) have detectable heart murmurs.

Dilated cardiomyopathy (choice D) is false as in this case there would be hypotension, tachypnea, tachycardia, gallop rhythm (S3 and/or S4), accentuated P-2, and murmurs of mitral and tricuspid regurgitation.

Page 113: Internal Medicine- Cardiology

Myocardial infarction (answer E) is false as the abrupt onset and the radiation of pain in this case is not a typical feature of MI. Also the serum creatinine kinase level is not elevated which is a cardinal feature for early diagnosis of MI. Fourth heart sound (S 4 ) may be heard in patients with ischemia.

A 72-year-old woman, who has had no major illnesses throughout her life, experiences 3 syncopal episodes over the past 2 weeks. In the last 2 days,

she has developed shortness of breath and cough with frothy white sputum.

On physical examination she is afebrile. Her blood pressure is 135/90 mm Hg. She has no peripheral edema. On auscultation, a systolic murmur is

detected. A chest radiograph reveals a prominent left heart border in the region of the left ventricle, but the other chambers do not appear to be

prominent. There is marked pulmonary edema. Laboratory studies show a total serum cholesterol of 170 mg/dL.

Which of the following is the most likely diagnosis?

1) Acute rheumatic fever 2) Mitral valve stenosis 3) Atherosclerotic aortic aneurysm 4) Calcific aortic stenosis 5) Infective endocarditis

The correct answer is choice D.

Senile calcific aortic stenosis involves gradual calcification of a an aortic valve and may not manifest itself clinically until there is narrowing of the outflow orifice to less than 1 square centimeter. Aortic valve disease can remain silent and then suddenly result in symptoms.

Major causes and predisposing conditions of aortic stenosis include acute rheumatic fever and bicuspid aortic valve. As individuals age, calcification of the aortic valve may occur and result in stenosis. This is especially likely to occur in people with a bicuspid aortic valve, but also occurs in the setting of perfectly normal valves as a result of age-induced 'wear and tear'. Typically, aortic stenosis due to calcification of a bicuspid valve manifests when individuals reach their 40s and 50s, whereas that due to calcification of a normal valve tends to manifest later, in the 70s and 80s.

The Triad of AS is Dyspnea on exertion, Angina and Syncope ("Aortic Stenosis is a S.A.D. story" may be a helpful mnemonic for remembering this triad). Aortic stenosis can cause dizziness, syncope and congestive heart failure. More symptoms indicate a worse prognosis. Treatment requires replacement

Page 114: Internal Medicine- Cardiology

of the diseased valve with an artificial heart valve.

In adults, significant aortic stenosis usually requires aortic valve replacement, which decreases the risk of death. According to a prospective, single-center, nonrandomized study of 25 patients, percutaneous implantation of an aortic valve prosthesis in high risk patients with aortic stenosis results in marked hemodynamic and clinical improvement when successfully completed.

Acute rheumatic fever (Choice A) is uncommon in individuals aged over 30 years. It is characterized by group A streptococcal pharyngitis, with clinical manifestations including chorea, carditis, subcutaneous nodules, erythema marginatum, and migratory polyarthritis.

Mitral stenosis (Choice B) is commonly caused by rheumatic fever. Features include a loud first heart sound, an opening snap, and a rumbling diastolic murmur.

Atheroslerotic degeneration (Choice C) of the arterial wall can result in the formation of an aneurysm. Aneurysms most commonly involve the abdominal aorta. Features of atherosclerosis, including aneurysmal involvement of other blood vessels may be present.

Classic features in infective endocarditis (Choice E) include bacteremia or fungemia, evidence of active valvulitis, peripheral emboli, and immunologic vascular phenomena.

A 66-year-old man admitted to smoking heavily for 45 years and presents with long standing exertional breathlessness. Clinical examination revealed

signs of emphysema, central cyanosis, a raised jugular venous pressure, and bilateral pitting leg oedema. Investigation showed cardiomegaly on chest X-

Ray and pulmonary function tests were consistent with a restrictive lung disease.

In Cor pulmonale, all of the followings are true, except:

1) May be acute or chronic 2) The first physical sign is usually a raised JVP 3) May be caused by sleep apnoea 4) The usual end result of many chronic debilitating lung diseases

Page 115: Internal Medicine- Cardiology

5) Is defined as right sided heart failure

The correct answer is Choice E

Cor pulmonale is right ventricular enlargement as a response to increased resistance or pulmonary hypertension in the lungs, mostly due to chronic obstructive lung disease (COPD). Typical signs include cyanosis, a raised jugular venous pressure, hepatomegaly and peripheral oedema. Chest X-Ray, ECG and Echocardiography will be sufficient to make a diagnosis.

Choice A Both can occur. Chronic cor pulmonale usually results in right ventricular hypertrophy whereas acute cor pulmonale usually results in dilatation of the right ventricle.

Choice B A raised jugular venous pressure is one of the first clinical signs, followed later by hepatomegaly and peripheral oedema.

Choice C Sleep apnoea can be associated with chronic hypoxaemia leading to pulmonary hypertension and Cor Pulmonale

Choice D Cor Pulmonale is usually secondary to chronic lung diseases. COPD, as mentioned is the most common cause, but others include:

Extensive loss of lung tissue due to surgery or trauma

Chronic, unresolved pulmonary emboli

Pulmonary veno-occlusive disorders

Page 116: Internal Medicine- Cardiology

Systemic sclerosis

Pulmonary interstitial fibrosis

Kyphoscoliosis

Obesity with alveolar hypoventilation

Neuromuscular disorders involving respiratory muscles

Idiopathic alveolar hypotension

Massive pulmonary embolism and acute Respiratory Distress Syndrome can cause acute Cor Pulmonale

Choice E It is not defined by right sided heart failure, but this is a consequence of chronic lung disease and pulmonary hypertension

Which of the following statement correctly represents the physical findings that distinguish the murmur of aortic stenosis from the murmur of hypertrophic cardiomyopathy?

1) The intensity of murmur of both AS and HCM decreases during strain phase of valsalva maneuver 2) The carotid upstroke in HCM is more brisk than AS 3) The intensity of the murmur of HCM increase in squatting 4) If a systolic thrill is present it is most often located in second intercostal space in HCM and at the apex in AS

Page 117: Internal Medicine- Cardiology

5) The murmurs of AS and HCM both radiate to carotid arteries

The correct answer is choice B

Both AS and HCM causes harsh systolic murmur but these conditions frequently can be differentiated by careful physical examination. The quality of carotid upstroke is different for both. In AS, outflow obstruction is fixed; carotid upstroke is delayed and diminished in amplitude. In HCM, carotid upstroke is brisk initially and diminishes in midsystole when the left ventricular outflow gradient becomes more pronounced. The pattern of radiation of murmur is also a distinguishing feature for these conditions. The systolic murmur of AS radiates to the carotid arteries but not the murmur of HCM (choice E).

If a systolic thrill is present, it is usually located in the second right intercostal space in AS whereas, in HCM it is more likely to be felt at the fourth left intercostals space (choice D). Beside these findings, dynamic bedside maneuvers are also helpful in distinguishing AS from HCM. Physiological maneuvers that enhance ventricular contractility increase the intensity of both murmurs. Maneuvers that reduce ventricular filling (eg: strain phase of valsalva maneuver or standing from squatting position) decreases the intensity of the murmur of AS, whereas decreased intraventricular volume causes LV dynamic outflow obstruction to intensify and the intensity of murmur of HCM increases (choice A). The murmur of HCM diminishes in intensity with maneuvers that augment LV filling, such as sudden squatting from standing position (choice C).

True statements about the release of cardiac enzymes due to acute myocardial infarction include all of the following, EXCEPT: 1) Normal subjects have undetectable level of plasma troponins 2) Most patients have troponin elevation by 2-3 hours after the onset of acute myocardial infarction 3) Troponins are preferred to CK-MB (MB isoenzyme of creatine kinase) due to their greater specificity and sensitivity 4) If troponin value is normal but the CK-MB is elevated, MB is most likely due to release from cardiac tissue 5) Reinfarction is diagnosed by an increase in the value of plasma troponins

The correct answer is Choice D

Page 118: Internal Medicine- Cardiology

The diagnosis of an acute myocardial infarction (AMI) has traditionally relied upon the combination of chest pain, electrocardiographic (ECG) manifestations, and elevations in serum or plasma biomarkers of cardiac injury. Chest symptoms are frequently atypical or absent and ECG abnormalities may be nonspecific or absent. As a result, the diagnosis of an AMI has increasingly depended upon evaluation of the rise of blood biomarkers. A variety of biomarkers have been used to evaluate patients with a suspected acute MI (troponins, MB-CK, myoglobin, lactate dehydrogenase). The cardiac troponins I and T as well as the MB isoenzyme of creatine kinase (CK-MB) are the most frequently used. Most normal subjects have undetectable level of plasma troponins (Choice A). It has also been shown that having a detectable level of troponin defines a group of stable patients who likely to have either significant coronary artery disease or elevated filling pressures. Cardiac troponin concentrations usually begin to rise two to three hours after the onset of AMI. Majority of patient (up to 80 percent) with AMI will have troponin elevations by two to three hours after presentation (Choice B). AMI can be excluded if there is no troponin elevation after 6-12 hours after onset. Troponins are preferred markers for the diagnosis of myocardial injury because of their increased specificity and better sensitivity compared to CK-MB (Choice C). At least one third of patients with an ACS have elevated troponins but normal CK-MB. If both are measured and the troponin value is normal but the CK-MB is elevated, MB is likely due to release from non-cardiac tissue (Choice D). Troponin elevations persist for one to two weeks after AMI, but values are usually not rising or falling rapidly at this time, allowing one to distinguish acute from more chronic events. Previously CK-MB was advocated to help diagnose reinfarction, but now troponin has subsumed that role. When reinfarction is suspected from clinical signs or symptoms following the initial infarction immediate measurement of troponin is recommended. Three to six hours later a second sample should be obtained. If there is a ≥20 percent increase in the troponin value of the second sample the recurrent infarction is diagnosed (Choice E).

A 19-year-old male, college student comes to the emergency department complaining of sharp chest pain progressively worsened over the past day. This sharp chest pain is central in location and radiates to the neck and trapezius ridge. The pain is worse with deep inspiration and lying flat; sitting forward seems to reduce the pain temporarily. One week previously, he had a self-limited upper respiratory infection. He denies any other medical history and takes no medications. Vital signs include a temperature of 37.9 C, blood pressure of 130/89 mm Hg, pulse rate of 100/min, respiratory rate of 14/min, and oxygen saturation of 98% on room air. ECG was also completed and shown below.

Page 119: Internal Medicine- Cardiology

Which of the following physical findings would be most specific in making the diagnosis:

1) S4 heart sound 2) Wide Pulse Pressure 3) Low grade fever 4) Pericardial friction rub 5) Kussmaul sign 6) A and B 7) B and D 8) D and E

The correct answer is choice D

This is a typical case of acute pericarditis. It is usually a young patient with sudden onset of sharp, dull, aching, burning, or pressing chest pain, occurring in the anterior chest and may be pleuritic in nature that is worsened with inspiration, due to concomitant pleural inflammation. The pain may be alleviated with sitting up and leaning forward while worsened with lying down, and also may radiate to the back, to one or both trapezius ridges. Fever may also be present.

Page 120: Internal Medicine- Cardiology

Acute pericarditis more common in men than in women. It is usually a short self-limited disease that lasts 1-3 weeks. On physical examination a pericardial friction rub is pathognomonic for acute pericarditis, the rub has a scratching, grating sound similar to leather rubbing against leather. Auscultation with the diaphragm of the stethoscope over the left lower sternal edge allows the best detection of the rub. Auscultation during end expiration with the patient sitting up and leaning forward increases the likelihood of observing this physical finding. Serial examinations may be necessary for detection. More than 50% of rubs are triphasic. They are composed of (1) an atrial systolic rub that precedes S1, (2) a ventricular systolic rub between S1 and S2 and coincident with the peak carotid pulse, and (3) an early diastolic rub after S2 (usually the faintest). The biphasic to-and-fro rub is less common (24%). It can occur with tachycardia and is due to summation of the atrial and early diastolic rub. Monophasic rubs (the ventricular systolic) are the least common but may occur in patients with atrial fibrillation. Especially when it is monophasic, the pericardial friction rub can be mistaken for a systolic murmur. Pericardial rubs may be differentiated if the rub does not change with usual respiratory or positional maneuvers, if 3 components are present, and if the findings on the ECG are typical. ECG finding include; diffuse, positive, ST elevations with reciprocal ST depression in aVR and V1.

Causes of acute pericarditis include:

1. Idiopathic Seasonal peaks occur in spring and fall

2. Viral Infection, viruses include coxsackievirus B, echovirus, adenoviruses, influenza A and B viruses, Enterovirus, mumps virus, Epstein-Barr virus, HIV

3. TB

4. Fungal organisms, Histoplasma, Blastomyces, Coccidioides

Page 121: Internal Medicine- Cardiology

5. Parasitic organisms, Entamoeba, Echinococcus, and Toxoplasma

6. Inflammatory disorders, Rheumatoid arthritis, Systemic lupus erythematosus, Scleroderma, Rheumatic fever

7. Metabolic disorders, Renal failure , Hypothyroidism, Cholesterol pericarditis

8. Cardiovascular diseases, Myocardial infarction, Dressler syndrome, Aortic dissection, Neoplasm

9. Miscellaneous causes, including iatrogenic; Drugs, Irradiation, Postpericardiotomy syndrome

10. Trauma

The S4 heart sound (choice A) is associated with any process that increases the stiffness of the ventricle: hypertrophy of the ventricle, long-standing hypertension (causes ventricular hypertrophy), aortic stenosis (causes ventricular hypertrophy), overloading of the ventricle (causes ventricular hypertrophy) fibrosis of the ventricle (e.g. post-MI) and Congestive Heart Failure.

Pulse pressure (choice B) is the difference between systolic and diastolic blood pressures. Normally, systolic pressure is about 40 mm Hg higher than diastolic pressure. Widened pulse pressure — a difference of more than 50 mm Hg — commonly occurs as a physiologic response to tachycardia, fever, hot weather, exercise, anxiety, anemia, or pregnancy. In this case, the pulse pressure is normal (30). Causes of wide pulse pressure include: atherosclerosis , chronic aortic regurgitation, thyrotoxicosis, patent ductus arteriosus, heart block, aortic dissection, endocarditis, raised intracranial pressure, vasodilating drugs, and Beriberi heart disease.

Page 122: Internal Medicine- Cardiology

Low grade fever (choice C) may be present, however, fever is not a specific finding of acute pericarditis.

Kussmaul sign (choice E) is a paradoxic rise in venous pressure with distension of the jugular veins during inspiration. This is due to late or chronic sequela of an inflammatory condition of the pericardium, like constrictive pericarditis, not in the acute phase as in the case of our patient.

Dr MUM